Of Particular Significance

Virtual Particles: What are they?

The term “virtual particle” is an endlessly confusing and confused subject for the layperson, and even for the non-expert scientist. I have read many books for laypeople (yes, I was a layperson once myself, and I remember, at the age of 16, reading about this stuff) and all of them talk about virtual particles and not one of them has ever made any sense to me. So I am going to try a different approach in explaining it to you.

The best way to approach this concept, I believe, is to forget you ever saw the word “particle” in the term. A virtual particle is not a particle at all. It refers precisely to a disturbance in a field that is not a particle. A particle is a nice, regular ripple in a field, one that can travel smoothly and effortlessly through space, like a clear tone of a bell moving through the air.  A “virtual particle”, generally, is a disturbance in a field that will never be found on its own, but instead is something that is caused by the presence of other particles, often of other fields.

Analogy time (and a very close one mathematically); think about a child’s swing.  If you give it a shove and let it go, it will swing back and forth with a time period that is always the same, no matter how hard was the initial shove you gave it.  This is the natural motion of the swing.  Now compare that regular, smooth,  constant back-and-forth motion to what would happen if you started giving the swing a shove many times during each of its back and forth swings.  Well, the swing would start jiggling around all over the place, in a very unnatural motion, and it would not swing smoothly at all.  The poor child on the swing would be furious at you, as you’d be making his or her ride very uncomfortable.  This unpleasant jiggling motion — this disturbance of the swing — is different from the swing’s natural and preferred back-and-forth regular motion just as a “virtual particle” disturbance is different from a real particle.  If something makes a real particle, that particle can go off on its own across space.  If something makes a disturbance, that disturbance will die away, or break apart, once its cause is gone.  So it’s not like a particle at all, and I wish we didn’t call it that.

Fig. 1: Two electrons approach each other; they generate a disturbance in the electromagnetic field (the photon field); this disturbance pushes them apart, and their paths are bent outward. One says they "exchange virtual photons", but this is just jargon.

For example, an electron is a real particle, a ripple in the electron field; you can hold one in your hand, so to speak; you can make a beam of them and send them across a room or inside an 20th century television set (a cathode-ray tube). A photon, too, is a real particle of light, a ripple in the electromagnetic field, and you can make a beam of photons (as in a laser.)  [Can’t have one in your hand though, since photons (in vacuum) are always moving.]

But if two electrons pass near each other, as in Figure 1, they will, because of their electric charge, disturb the electromagnetic field, sometimes called the photon field because its ripples are photons. That disturbance, sketched whimsically in green in the figure, is not a photon. It isn’t a ripple moving at the speed of light; in general isn’t a ripple at all, and certainly it is under no obligation to move at any one speed. That said, it is not at all mysterious; it is something whose details, if we know the initial motions of the electrons, can be calculated easily. Exactly the same equations that tell us about photons also tell us about how these disturbances work; in fact, the equations of quantum fields guarantee that if nature can have photons, it can have these disturbances too. Perhaps unfortunately, this type of disturbance, whose details can vary widely, was given the name “virtual particle” for historical reasons, which makes it sound both more mysterious, and more particle-like, than is necessary.  [Students of math and physics will recognize real photons as solutions of a wave equation, and virtual photons as related to the Green function associated with this equation.]

Fig. 2: As in Figure 1, for a positron (an anti-electron) and an electron; now the slightly different disturbance causes the two particles to attract one another, and their paths are bent inward.

This disturbance is important, because the force that the two electrons exert on each other — the repulsive electric force between the two particles of the same electric charge — is generated by this disturbance. (The same is true if an electron and a positron pass near each other, as in Figure 2; the disturbance in this case is similar in type but different in its details, with the result that the oppositely charged electron and positron are attracted to each other.) Physicists often say, and laypersons’ books repeat, that the two electrons exchange virtual photons. But those are just words, and they lead to many confusions if you start imagining this word “exchange” as meaning that the electrons are tossing photons back and forth as two children might toss a ball. It’s not hard to imagine that throwing balls back and forth might generate a repulsion, but how could it generate an attractive force? The problem here is that the intuition that arises from the word “exchange” simply has too many flaws. To really understand this you need a small amount of math, but zero math is unfortunately not enough. It is better, I think, for the layperson to understand that the electromagnetic field is disturbed in some way, ignore the term “virtual photons” which actually is more confusing than enlightening, and trust that a calculation has to be done to figure out how the disturbance produced by the two electrons leads to their being repelled from one another, while the disturbance between an electron and a positron is different enough to cause attraction.

Fig. 3: An electron may naively be thought of as a ripple of minimum intensity --- the minimal ripple --- in an electron field. But the electron interacts with the photon field (i.e. the electromagnetic field) and can create a disturbance in it; in doing so it too ceases to be a normal particle and becomes a more general disturbance. The combination of the two disturbances (i.e. the two "virtual particles") remains a particle with the energy, momentum and mass of the incoming electron.

Now there are many other types of disturbances that fields can exhibit that are not particles. Another example, and scientifically one of the most important, shows up in the very nature of particles themselves.  A particle is not as simple as I have naively described. Even to say a particle like an electron is a ripple purely in the electron field is an approximate statement, and sometimes the fact that it is not exactly true matters.

It turns out that since electrons carry electric charge, their very presence disturbs the electromagnetic field around them, and so electrons spend some of their time as a combination of two disturbances, one in in the electron field and one in the electromagnetic field. The disturbance in the electron field is not an electron particle, and the disturbance in the photon field is not a photon particle.  However, the combination of the two is just such as to be a nice ripple, with a well-defined energy and momentum, and with an electron’s mass.  This is sketchily illustrated in Figure 3.

Fig. 4: The Feynman diagram needed to calculate the process in Fig. 3. One says "the electron emits and reabsorbs a virtual photon", but this is just shorthand for the physics shown in Fig. 3.

The language physicists use in describing this is the following: “The electron can turn into a virtual photon and a virtual electron, which then turn back into a real electron.” And they draw a Feynman diagram that looks like Figure 4. But what they really mean is what I have just described in the previous paragraph. The Feynman diagram is actually a calculational tool, not a picture of the physical phenomenon; if you want to calculate how big this effect is, you take that diagram , translate it into a mathematical expression according to Feynman’s rules, set to work for a little while with some paper and pen, and soon obtain the answer.

Fig. 5: As in Figure 3, for a photon. The photon can become a disturbance in the electron field. This disturbance has some regions with negative electric charge and some with positive electric charge, but with total charge zero, like the incoming photon itself. The photon can do the same with other charged fields, such as the muon field.

Another example involves the photon itself. It is not merely a ripple in the electromagnetic field, but spends some of its time as an electron field disturbance, such that the combination remains a massless particle. The language here is to say that a photon can turn into a virtual electron and a virtual positron, and back again; but again, what this really means is that the electron field is disturbed by the photon. But why are we seeing a positron — an anti-electron — and yet I am only referring to the electron field? The reason ties back to the very reason that there are anti-particles in the first place: every field, by its very nature, has particle ripples and anti-particle ripples. For some fields (such as the photon field and Z field) these particle and anti-particle ripples are actually the same thing; but for fields like electrons and quarks, the particles and anti-particles are quite different. So what happens when the electron field is disturbed by a passing photon is that a disturbance is set up that has some electron-like disturbance with net negative electric charge, and some positron-like disturbance with net positive charge, but the disturbance as a whole, like the photon itself, carries no net charge at all.

For those who learned (and recall a bit of) freshman physics, what is happening is that the oscillating electric field that makes up the photon is polarizing the electron field — inducing a dipole moment. Remember dielectrics and how electric fields can polarize them? Well, the vacuum of empty space itself, because it has an electron field in it,  is a polarizable medium — a dielectric of sorts.

Fig. 6: The Feynman diagram needed to calculate the process in Fig. 5. One says "photon becomes a virtual electron-positron pair", but this is just shorthand for the physics shown in Fig. 5.

The same is true, by the way, for all the other electrically charged fields, including those of the muon, the up quark, and so forth.

[Here, by the way, we come across another reason why “virtual particle” is a problematic term.  I have had several people ask me something like this: “ Since the diagram in Figure 6 seems to show that the photon spends some of its time as made from two massive particles [recall the electron and the positron both have the same mass, corresponding to a mass-energy (E = m c-squared) of 0.000511 GeV], why doesn’t that give the photon a mass?”  Part of the answer is that the diagram does not show that the photon spends part of its time as made from two massive particles.  Virtual particles, which are what appear in the loop in that diagram, are not particles.  They are not nice ripples, but more general disturbances.  And only particles have the expected relation between their energy, momentum and mass; the more general disturbances do not satisfy these relations.  So your intuition is simply misled by misreading the diagram.  Instead, one has to do a real computation of the effect of these disturbances.  In the case of the photon, it turns out the effect of this process on the photon mass is exactly zero.]

Fig. 7: The electron can generate disturbances in the photon field; the resulting photon disturbance can in turn create disturbances in other electrically charged fields, such as the muon field.

And it goes on from there. Our picture of an electron in Figure 3 was itself still too naive, because the photon disturbance around the electron itself disturbs the muon field, polarizing it in its turn. This is shown in Figure 7, and the corresponding Feynman diagram is shown in Figure 8.  This goes on and on, with a ripple in any field disturbing, to a greater or lesser degree, all of the fields with which it directly or even indirectly has an interaction.

Fig. 8: The Feynman diagram needed to calculate the process shown in Figure 7.

So we learn that particles are just not simple objects, and although I often naively describe them as simple ripples in a single field, that’s not exactly true.  Only in a world with no forces — with no interactions among particles at all — are particles merely ripples in a single field!  Sometimes these complications don’t matter, and we can ignore them.   But sometimes these complications are central, so we always have to remember they are there.

515 Responses

  1. Okay I clearly don’t know a lot of physics and even less quantum physics, but is it really fair to say that virtual particles are not particles and the Feynman diagram is not a “picture” of the physical phenomena because for any physical phenomena to be understood we have to, in some sense, imagine it in a way that is useful? Such as how the kinetic theory of matter imagines particles to be tiny balls that vibrate and transfer energy.

    Although this article claims that the disturbances in the field and the math behind it is the real picture of the phenomena it just replaces one abstraction with perhaps a more fundamental abstraction. But even those diagrams can ultimately be replaced by just pure mathematical rules and objects that define the physical phenomena at the most abstract level. I didn’t understand why this would mean the less abstract way to see the thing is not the real picture. Because even the quantum fields also “exist” and are real “pictures” in the sense that they are a visual and mathematically useful to predict quantum phenomena.

    1. Thanks for this tricky and thoughtful question. You do raise important issues about what it means to “envision” or “picture”, and what understanding best consists of.

      My goal, as a scientist and as a teacher, is the following. It is to develop a “picture” — meaning both a conceptual one, a verbal one, and, to the extent possible, a visual one — of the phenomena that matches the phenomena themselves. In other words, one conceptual “picture” is better than another if the former correctly matches what nature actually does in a wider range of situations. Successful mirroring of a physical phenomenon within one’s brain is, after all, a key measure of understanding.

      So what I am really saying here, from that point of view, is that the “virtual particle” language promulgated widely does a poor job of matching the processes of nature, whereas quantum field theory’s own equations can be expressed in a much more accurate language that matches the phenomena better.

      In short, the best approach to understanding, in my view, is to start with the successful map between experiment and math — between particle physics observations and the mathematics of quantum field theory which mirror them spectacularly well — and continue onward by carefully mapping between the math and the conceptual, verbal and pictorial ideas that we use to think about and talk about nature. If the concepts mirror the math, and the math mirrors the experiments, then we have a relatively high level of understanding.

      By contrast, if our language directly contradicts the math, then our understanding is worse. And the language of “virtual particles” causes people to make all sorts of incorrect statements. As an example, there have been statements in the media that “there are charm quarks in the proton”, a statement which, were it true, would violate energy conservation and E=mc^2. Trying to explain it in terms of “virtual particles” is extremely confusing, because it makes it sound as though energy conservation is violated (briefly) by quantum physics, whereas the math shows that this is absolutely wrong: that energy conservation is never violated. To clarify the issues, I wrote this post, and there you will see evidence that these issues are neither minor nor academic.

      I hope that clarifies why I took this point of view. Happy to pursue the question further if I have not really answered you.

  2. Although more technical for your intended audience, this post was discussed on Physics Stack Exchange in 2016, a popular physics forum via the question: “Can virtual particles be thought of as off-shell Fourier components of a field?”

  3. This article offers the BEST explanation I have been looking for many months. I finally understand.

  4. Professor Strassler,

    Thank you for this explanation. I’ve had a life long appreciation of science but no formal education beyond high school and college general education requirements. I’ve been generally aware of virtual particles for years (as explained by pop science). But the last couple weeks I’ve wanted to dig deeper and stumbled upon various explanations of virtual particles across the web and was left more confused than when I started. I found your post to be very helpful, so I just wanted to thank you.

      1. I did check it out and thoroughly enjoyed it also, thank you. I’ve found your website to be a great resource and very eye opening. I won’t pretend to understand the math involved, even a little. But your explanations have been very illuminating.

  5. Thanks for the answer. Kind of throws a big kink into my theory of time dilation, gravity and dark matter.

    1. Good for you for recognizing that. An important lesson for every physicist, especially amateurs: never, ever let your love for a theory blind you to the facts of experiment.

      There are a million wrong paths, and a million ways to take each one, if you don’t learn what the full range of experiments have told us about nature. It’s the extreme tyranny of experiments which explains why not a single professional theoretical physicist I know has a personal “theory of everything.” It’s just too hard to be consistent with all the experiments at once. Even Einstein didn’t manage it.

      1. I was sitting with Renate Loll during lunch at the Quantum Gravity conference in Vancouver a couple of weeks ago. She asked about my theory and it was all fine until she said “what about glueballs?”

  6. Your question is ill-posed, so please try to ask it again, being more precise if you can. To that end, note that protons are color-neutral and are not surrounded by a non-zero gluon field (just as electrically neutral neutrons and electrically neutral hydrogen are not surrounded by a non-zero electric field.) I’m not sure whether you were assuming this or the reverse. Also note that gluons are ripples in the chromoelectromagnetic field. You seemed to be asking whether gluons *generate* such ripples, but then you said the ripples were virtual particles, which they are not. So altogether I’m not sure what you’re asking.

    1. A bar magnet is electrically neutral, but still generates a strong magnet field surrounding the magnet, once the free electrons are polarized. We even know that a living frog (also electrically neutral) has a magnetic moment. I would imagine that a large number of gluons moving around at c would also generate a significant disturbance in the chromo field surrounding the proton. That’s what I’m asking.

      1. Very good; that’s clear. The big difference between electromagnetism and photons vs
        “chromodynamics” and gluons is that electric flux and magnetic flux are free to expand outward (creating a substantial electric field around a electrically polarized molecule, similar to your magnet example) while chromoelectric flux is trapped, or “confined”. So yes, there are enormous fields *inside* the proton, but the edge of the proton is set by this trapping effect, and the fields cut off very rapidly at the edge. If there were no trapping effect, the proton would be much larger, comparable to (hbar/c) times 1/masses of the quarks or even larger, and the whole system would be very different.

        see also: https://profmattstrassler.com/articles-and-posts/particle-physics-basics/the-known-apparently-elementary-particles/jets-the-manifestation-of-quarks-and-gluons/ , https://profmattstrassler.com/2018/02/05/a-brilliant-light-disappears-over-the-horizon-in-memory-of-joe-polchinski/

        A similar effect, but flipping electric effects with magnetic effects and vice versa, is seen in a superconductor. If you put a bar magnet inside a superconductor, the magnetic field would not expand out the way it does in vacuum. It would run in a one or more little tubes around the magnet, called Abrikosov vortices. https://en.wikipedia.org/wiki/Abrikosov_vortex

        So: Magnetic flux is confined i.e. trapped within superconductors, and gluon-field flux is confined in our universe’s vacuum for roughly analogous reasons. The precise reasons are not known; hence the million dollar prize for a proof: https://www.claymath.org/millennium-problems . [There are some similar systems where quasi-proofs have been given; I’m partly responsible for a couple of the recent ones; Seiberg and Witten gave one of the first in the early 1990s. But the real-world case hasn’t yet been done.]

        By the way, the the tubes of chromoelectric flux that would be present if no quarks had low masses — analogous to the Abrikosov vortices — would behave as strings. This is why theories of gluons are closely related to string theory, and in some cases that relation is precisely known, though again, not for the real world case or its cousin without light quarks.

  7. Thanks very much for this. Although gluons are confined, is it correct that they would still generate ripples (virtual particles) in the gluon (chromoelectric) field surrounding the protons in which they are confined? And if so, would these ripples spread out 1/r^2 from a planet?

  8. Matt, thanks for these posts.

    In the following statement, what do you mean by the term “generally?”

    A “virtual particle”, generally, is a disturbance in a field that will never be found on its own, but instead is something that is caused by the presence of other particles, often of other fields.

    1. Good question.

      The reason for the weasel word “generally” is that when forces are relatively weak, it’s easy to say what a virtual particle is and isn’t (using, for instance, the language of Feynman diagrams), but when they become strong (as inside a proton, where Feynman diagrams are useless) it becomes increasingly ambiguous.

      For physicists, this ambiguity doesn’t matter; it causes no technical problems, and we simply don’t need to specify whether a disturbance is a real particle or a more general virtual disturbance. It really only matters for the language we use and for the way we explain these concepts to students and non-experts. As an example, when I say a virtual particle is “something that is caused by the presence of other particles”, that’s a statement which is clearly true for weak forces, but somewhat ambiguous when forces are strong.

      Clearly a subtlety like this belongs in a more advanced article. I didn’t want to get into it within a basic article on the “virtual particle” concept, so I used a weasel word.

  9. Professor Strassler: Would love to see your explanation of “Action at a Distance” of entangled particles; “The EPR Experiment”. I have read about this a lot but do not understand it. If you know that the 2 particles are “entangled” (whatever that means; I assume they interact with each other and say have the same spin) then if you know the property of one of course you would know the property of the other, wherever it is. Why is this such a big deal? Thanks so much.

  10. When a free electron scatters from an atom, is it correct to think of this as an exchange of virtual quanta between the free electron and the bound electrons and protons in the atom? If that is the case, how does one calculate the spectrum of virtual quanta exchanged during the collision?

  11. This is a great and super helpful article although I am admittedly about a decade late to the game so I have no clue if you will see this. Anywho, I’m writing about virtual particles for a non-scientific essay and wanted to check the correctness of my definition. Seeing as I don’t know anyone with knowledge of quantum science irl, this seemed like an okay resort. It’s meant to be incredibly basic, really just enough for it to be not inherently incorrect.
    “Quantum physicists discovered anomalies in the ripple of particles within their field. The disturbance causes such intense ripples that the particles will sometimes dip into other surrounding fields. At first glance, it appears that these particles are popping in and out of existence when in truth they are colliding with different regions of reality – this disturbance is referred to as virtual particles.”
    Thanks! I’ll take any help I can get

  12. USE THE DIFFERNCE IN THE MOON’S GRAVITY AS A TOOL AT HIGH AND LOW TIDE AND COUNT THE AMOUNT OF PHOTON PRODUCTION IN A VACUUM.

  13. Sosyal medya uygulamalarının en popüler uygulaması tiktok için hemen takipçi satın al. Tiktok ucuz takipçi satın almak için websitemizi ziyaret et ve hemen en ucuz takipçi satın al.

  14. [An example of a quantum field theory that has no particles is a “conformal field theory”, very important in studying matter at a phase transition, such as a magnet at the critical temperature where it loses its magnetization.]

  15. Hello
    think the real issue with virtual particles is that quantum dynamics is subject to a massive ambiguity (i.e. we have initial and final states and no information about what actually goes on during the evolution between these states – this is really what the path integral is telling us).

    1. Quantum theory says that information is always lost. No information exists between the initial and final states because nothing is going on except a time delay. The problem is most physicists are mechanists first and scientists only if it doesn’t get in the way of mechanism.

  16. The article generates an easily digestible image of what’s going on in an interaction. But I’m still confused – having to rely on highschool knowledge and an (probably misleading) analogy of water waves – my picture of excitations moving across each other is the superpositions of the 2 but with no mutual effect – once they’ve passed each other they remain intact. In your description is sounds like the ripples that occur as an effect of the superposition do have an effect. Obviously, I missed something fundamental .. Many thanxs in advance for getting me on the right track

  17. Want to go a step further?
    What if there are no particles, but only fields..

    Here is the Loader theory as invented by Max Planck,
    which has been verified with various laboratory experiments.
    https://saidit.net/s/Physics/comments/18rz/nullhypothesis_quantum_mechanics_shows_that_there/
    I would love to get some feedback on the implications or validity.

    I call it the null-hypothesis for quantum mechanics,
    because it completely breaks with the particle model of light.
    Personally not my favourite, but it is scientifically the best choice.
    Because it is confirmed in the observations and it is the most simple one.

    1. Virtual particles are not observable but their effects are. They are part of a model that has a great deal of explanatory value but is an abstraction that may or may not be real in the ordinary sense. It is important not to try to visualize the model rather it should be used to make predictions.

  18. Virtual particles are the creation of a circular argument. They were invented by physicists to cover the holes in their theories. They are mythical creatures to use to explain what the physicists do not know.

  19. I think the real issue with virtual particles is that quantum dynamics is subject to a massive ambiguity (i.e. we have initial and final states and no information about what actually goes on during the evolution between these states – this is really what the path integral is telling us). So while talking about disturbances in quantum fields is fine, virtual particles is just one way of mathematically describing these (and there may be many). So as you say, there may be something real to be had from quantum field theory (though I am not sure special relativity should be part of fundamental reality – I also wonder if fields aren’t just convenient mathematical objects rather than being particularly close to reality) virtual particles are just a mathematical convenience, although they may represent things of real significance.

  20. Great article. Thanks. I have been studying QFT for some time but such articles help me in seeing the actual picture that mathematics is trying to portray. This article also killed many misconceptions that I had picked up from some layperson books.

  21. Professor Matt Strassler: I just saw this article. I am completely convinced and agree with you. I wonder if you are still active on this site? I have a question regarding how to compute the attraction or repulsion of two particles. For example, Fig. 1 says: “Two electrons approach each other; they generate a disturbance in the electromagnetic field (the photon field); this disturbance pushes them apart, and their paths are bent outward.” Can we mathematically prove this? Is there a scheme to compute the disturbance in the photon field and the paths of these two electrons, and mathematically prove that they repel each other?

  22. What determines whether the outcome particle has to be a photon or an electron when the same given input photon & electron fields interact with each other? Also what makes certain regions of the field different from the others such that only these regions of the field upon interaction with certain regions of another field become particles?

  23. So is it possible that the Higgs mechanism is the same thing as (or part of) a particle’s self energy? There is no physical Higgs field spread through space. A particle’s ejecting and absorbing a weak charge continuously is the same as the photon sometimes becoming an electron.

  24. I started reading 3 hours ago, and I’m still here! 🙂 Great, great reading. Thanks to everyone!

  25. Thanks for the good visualizations.. So is the short version that u think that because the virtual systems dont move far without changing, they are illusions of a deeper truth? Hm ok.. But do any particles travel any distance without them? They seem pretty inseparable? Particles and the virtualizations.. and sepreating them, lets say at the planch scale, seems basicly meaningless and maybe even impossible? Dont some units, maybe all units, spend some time as very differant units on their way.. i mean if all units are virtual at least at some point and that seem almost certian especially if one considers the tools mesurement “observer” issues.. my favorit example of virtual particles is fission, those neutons dont have to be anywhere near that fat nuclei to nock it! Oh that and halo nuclei.. is it a mesonic/gluonic tube giving them lifelines to orbit with?.. who knows.. but it all seems pretty real and particlely enuough to me..

  26. Sir , Does the presence of virtual particles in the vacuum bend space-time fabric and create gravity ?

  27. Hi Professor Strassler,

    Thanks for the explanation of virtual particles, it really cleared up a lot of misconceptions that I had.

    The big takeaway I got from the explanation is that 1) virtual particles (or “disturbances” in the field) are real things, just not real particles, and 2) these disturbances must be discrete, since they occur in a quantized field.

    If you don’t worry for the moment about how you describe the disturbances (with Feynman diagrams or whatever) the fact that they are quantized means that, for instance, the Compton scattering of two electrons will transfer momentum in discrete steps, corresponding to the discrete virtual particles (or disturbances) that interact with them.

    I haven’t found anything in the literature that characterizes this process at all. If the interactions are discrete they must have some properties, such as how many of them, how often they occur, what is their energy is, etc. Maybe I’m just not looking in the right place.

    It also seems to me that since changes in the momentum of particles (what we call “acceleration”) must happen in discrete steps, this is at odds with General Relativity, where acceleration is continuous. It seems to me that if we could reconcile these two views of acceleration, we’d be a lot closer to the Grand Unified Theory.

  28. if electromagnetic waves can travel through vacuum, it means that vacuum actually contains (or is made up of) electric fields and the waves are just disturbances of that field. Gravity is also a disturbance in spacetime caused by masses. Doesn’t this create a symmetry between the two forces since both gravitation and electromagnetic are actually intrinsic to the nature of vacuum itself?
    Some say that EM fields are not intrinsic to spacetime and are hence different from gravitation in this regard, but how can we prove that?

  29. What about virtual particles being divided by a black hole? It’s partner escapes while one enters the hole. It also causes the black hole to disapate. How does one particle escape a black hole? Confused about what is happening in the event horizon.

  30. Thank you that was very helpful. I am absolutely a late person I am a plumber with a high school education. I took no Sciences Beyond basic biology and paid hardly any attention. But I have been interested in science since I was 4 years old. I was wondering what you thought of the quantum field and do they exist before the Bigbang. I also wondered do you think the fields exist at the singularity. Thank you again

  31. Mat, you are a genius. I did not understand everything, but I am left with hope that what you said makes sense, even intuitively 😀

  32. A careful use of language as illustrated by the author of this article clarifies a subject I’ve been interested in for many years : a mental miracle!

  33. Consider whether or not a quantum fluctuation conserves or not energy, linear momentum, angular momentum, electric charge, baryon number,…. and for how long the fluctuation lasts. Then you will be able to answer your own question.

    1. I have no doubt as to the validity of Heisenberg’s uncertainty principle regarding quantum fluctuations and the energy and time product, and the popping into and out of existence of particles in free space. . That was not the crux of the question. The question was whether the uncertainty principle is required to explain forces, being transmitted by a field that I pumped energy into.

      Particles in free space popping into and out of existence, with no apparent source of energy, is one thing. Real particles coming into existence, from a high energy collision, that I pumped energy into is another.

  34. Professor Strassler:
    This was the best explanation of virtual particles I have ever read. If I had two positive charged particles, that I “force” closer together, and then pin them in place. The “work” I did forcing them together appears as energy in the field between the particles. Now, it is often explained that the forces being felt by these particles is being mediated by “virtual particles”. So far so good. I understand that these “virtual particles” are not really particles, as explained in your awesome article. But, why do they say that virtual particles temporarily violate energy conservation? The energy is IN the field, why/ how is energy conservation being violated?

    1. Heisenberg Uncertainty Principle in Energy and Time: (DeltaE)(DeltaT)(greater or equal)(Planck’sConstant)/(2 Phi)

      1. Yes, I’m aware of the Heisenberg’s Uncertainty Principle. But, if I have put the energy into the field, why is a violation of conservation of energy required for “virtual particles” ( which are not particles). If the energy is uncertain that’s fine, but how is that a violation of conservation of energy?

        1. Uncertain means not well defined. How can you conserve something not well defined? I presume the Uncertainty Principle means energy only approximately conserved in the Universe from time of beginning until now because (deltatime) not infinite.

          1. Well, a lot of people say that, but I don’t buy it.
            The tend to say such things in situations where you
            cannot measure the energy (due to fundamental
            uncertainty issues).

            So, in a situation where the energy can’t be measured,
            what would you prefer?
            a) assume it isn’t conserved, or
            b) assume it is conserved.

            Dr. Strassler made a comment about this a few lines up, “a white lie” that I was hoping he can elaborate on:

          2. The work I did forcing the two positive charges together appears as energy in the field. If the original intent of “virtual particles” was that you needed additional energy to create these particles, and the energy did not come from the energy content of the field, then I can see a violation of conservation of energy. However, as the article states, “virtual particles” are not particles, they are a disturbance of the field, as such it would seem plausible that the energy put into the field, by my work, is the disturbance itself.

  35. Great explanation. I’m reading a layperson’s book right now, which is very good, but this really clears it up.

  36. This is mind-calming clarity, thank you very much. Not that everything went down smoothly, but at least it is nice to see that it HAS a rigorous definition despite its ill-fitting name. In my head I always thought virtual particles were events or effects that behave like particles, and can be calculated as particles – but this doesn’t explain anything, it is just an explanation that avoids explaining. A lie-to-children of sorts, to myself.

  37. Does anyone have a device or ability to detect & measure gravitational anomalies? I have an unusual thought, depending on the answer to the question above, and believe that you may find it intriguing!

  38. Dear Prof. Strassler. I want to use the second para. in your article in a book to be self-published shortly but haven’t been able to get rights permission on a fair use basis, and so have paraphrased. But your own words would be much better. Would you kindly give me fair use permission, provided of course I provide a suitable citation? Yours, Terry Higham

  39. Hello Professor Strassler,
    Thank you for the excellent blog on virtual particles.
    However, maybe I’m missing something, but I was hoping to find a way that these field disturbances might explain the 1/r dependence of the usual Coulomb electric potential. Could it be that the field disturbance amplitudes that you discuss here usually drop off as 1/r, where r is the distance from the charged particle? It seems to me that an explanation, that is palatable to the lay person, of the 1/r dependence of both charged particles electric potentials and massive particles gravitational potentials would a very important product of all of this virtual particle or field disturbance model that is being discussed.
    Thanks in advance.

  40. Good morning professor Strassler and thank you for your interesting blog.
    In the comment of March 21st 2012 to Dr HW Looi you told there are many theories as to how space comes to be, including theories that have time but not space at first, and then develop space through a very subtle mechanism.
    May you suggest me something to read about these theories and mechanisms ?
    Many thanks in advance and best regards

  41. In other words I mean to say transformation (creation/anihiliation) of particles is itself mediated by something formless/dimensionless/timeless at particle level too, needless to perceive bigbang for existance of such something or occurence such one-time transition/transformation-from not-being to being as there is no such thing not-being it is subtle-being. Someone may call it mystic interpretation, but physics seems to be heading towards it.

  42. Manifestation of Unmenifested (formless/dimensionless/timeless) to Menifested and vice versa: Transition (possibly logically there should be otherwise quark/neutrino should not change its flavor on the fly) from formless/dimensionless/timeless particle (if one like to call it a particle at all) with spacetime and energy/matter combined to elementary particles may occur in so many different (may be infinite) ways with different particles having different form (charge etc), dimensions (quantum states) and lifetime. But still it is an indicator as of now, let us wait for another year or so, meanwhile read:

    http://www.nature.com/news/physicists-excited-by-latest-lhc-anomaly-1.21865

    Statistical confidence (guesswork on behaviour of nature) is one thing way of working of nature is something different.

  43. I enjoyed your article about “virtual particles”. I have studied physics but not particle physics and one question I have always had relates to the Heisenberg Uncertainty Principle. In freshman physics texts the Heisenberg Uncertainty Principle is used to estimate the mass of pions. And then in the same texts it is often shown that the “fluctuation energy” as calculated from the Uncertainty Principle can be vastly different from the “disintegration enegry” i.e. the energy that results if one calculate m c^2 using the rest mass of the pion. This has always seems paradoxical to me.

  44. For example, an electron is a real particle, a ripple in the electron field; you can hold one in your hand, so to speak; you can make a beam of them and send them across a room or inside an 20th century television set (a cathode-ray tube). A photon, too, is a real particle of light, a ripple in the electromagnetic field, and you can make a beam of photons (as in a laser.) nick

  45. Patterns in space (crystals) so far and now we are going to see patterns in time (Pls visit the Web link in my above post), so patterns in spacetime-matter combined (compressed elements what I use to say) may be our logical line of thoughts in solving black holes, dark matter, dark energy like issues as explained in my above posts.

  46. http://www.sciencealert.com/physicists-calculate-long-lost-signals-emitted-fractions-of-a-second-after-the-big-bang?perpetual=yes&limitstart=1

    The above link makes my above posts a bit more interesting while leaving the questions open that why cant super strong gravitational waves inside blackholes compress spacetime along with matter into something (may be dark matter-anihiliation of which results in dark energy, matter and radiation) lasting for longer durations and why inflaton cannot occur again and again? My assumption is only that the contraction/expansion of spacetime alongwith matter is already built in, thus no need to assume one time bigbang. Is this a reasonable assumption???????…If yes, by the way this assumption may lead to existance of hyper spacetime along with hyper matter possibly consisting of dark matter.

  47. The crux of my comments posted above is contraction and expansion of particles along with space particles in discrete scene to below Planck constants and other limits (where these become inapplicable or not obeyed) and to Planck constants and other limits ( where these become applicable or obeyed). Since we are talking about something which is covered somehow under uncertainty princpile then the requirements of extraordinary evidences or mathematical rigor not becomes so stringent or even not possible in my opinion as it is an assumption not a claim and we have to see whether it is a reasonable assumption or not and we have to see that if such reasonable assumption leads us to better understanding of Nature under one framework/theory. Learned readers are always free to make their opinion on merits/demerits of my comments.

  48. If the experiments do not find any dark matter particles interacting with normal particles in next 5-10 years then it will simply means that dark matter particles are not obeying planck constant and other limits and like wave particle duality in uncertainty principle there won’t be any means to know how dark matter particles interact with normal matter, matter is bound to stay as dark matter or normal matter with no intermediate interacting state observable to us like in between wave and particle or at the most dark matter particles may appear as vacuum energy.
    One question is repeatedly occurring to my mind is that, if space is discrete then how space particles trapped in atoms and protons etc behave in case of moving particles, whether these space particles move along like inside air molecules moving along with flying aircraft or outside air molecules not moving along with the flying aircraft? What will be the physics as far as mass and gravity are concerned if trapped space particles are moving against space particles as mentioned in first part of the previous sentence?

  49. …To continue further, mere presence of black holes is quite indicative of existence of substance inside black holes that do no obey planck constant and other limits at micro level. Same is true when we talk about dark energy or matter that means we are not sure about “the dark” form of energy or matter which should have obeyed planck constant and other limits to make the things to be understood in our normal discourse of physical understanding of nature. Uncertainty principle is a sham which has revealed to as well as hide from us considerably. The appendage of black, dark, vacuum or virtual seems very straight forward with all optimism we want to have for future discoveries that along with division of universe in observable and unobservable at cosmological level, division of observable and unobservable particles (obeying planck constants and limits or not) at quantum level is going to stay here and may be it is an extended form of uncertainty principle. By leaving “came into existance out of nothing concept” aside, to me, unless we try to bridge the gap by allowing everexisting expandable and contractable universe and particles certainly there won’t be significant level of increase in our understanding of nature under single framework/theory.

  50. Few points, though I cannot say proofs, are quite indicative of existence of virtual particles in collapsed/compressed state forming base of single framework:
    1. Existence of massive galaxies with abundance of dark matter and with relatively not-so-massive (small) black holes at their centers: can be inferred as dark matter containing collapsed particles are produced by none other than black holes by compressing particles and fuzzy space particles into compressed particles.
    2. Release of coherent high energy photons (xray) by black holes: can be inferred as on going process of compression of particles and fuzzy space particles into compressed particles in coherent fashion.
    3. Release of polarised CMB-an inverse process of point #2, into particles and fuzzy space particles appearing as dark energy. CMB with redshift by taking into account velocity associated with earth, solar system, milky way galaxy, great attractor and so on… is not an absolute proof of occurence of big bang.
    4. No direct interaction of dark matter with normal matter, except through gravity, can be safely inferred as dark matter is not obeying planck constant and limits.
    5. Besides above I personally believe that technical advancement in next 5-10 years will dispel our present belief regarding age of universe and bing bang model. We are sooner than later going to find much older galaxies than the present estimated age of universe with super massive black holes at their centers disproving big bang model ab initio. Hence expanding and collapsing universe in cyclic local fashion along with expandable and collapsible particles will gain attraction.

  51. I personally agree with above “pingback” that all particles are virtual till these become observable. I also like to repeat that we need to allow virtual particles remain beyond planck constants and limits under collapsed state till these become observable under expanded state obeying planck constants and limits. Expanded (observable) and collapsed (virtual) particles, to me, seems to be key to understanding black holes, dark matter, dark energy, vacuum energy, wave-particle duality, uncertainty principle and discreteness of spacetime under single framework and gravity will be automatically linked to quantum world via inverted gravity.

  52. Sorry for the typing mistakes in my above posts, the text of these posts was typed using small cell phone, kindly bear the inconvenience. Thanks.

  53. And to move my hypothetical scenario further, I would like to add that my hypothetical fuzzy particle/zero point element, as posted earlier today, very very very tiny mass, very very very tiny momentum, very very very tiny charge and zero spin. To repeat it , these fuzzy particles are produced by combining/compressing normal matter particles and space particles into one in the centers of black holes in some coherent fashion. In doing so these fuzzy particles (candidate for dark matter) do not violate conservation laws as though these have tiny mass, momentum and charge and rest of the conserved energy as inverted gravity (short distances repulsive anti gravity with little affect at long distances), due to inverted gravity these fuzzy particles collectively in large numbers travel out of centers of black holes defying the blackhole gravity and spread out without cusp-halo problem.
    I think my posts are very good subject for a science fiction at least. Rest depends upon learned community to accept these or not. And I also do not expect that these questions will have any answer in near future though I will keep on trying to put dark energy, dark matter, black holes, vacuum energy, expansion/contraction of universe etc etc under one framework.

  54. Professor Strassler:

    Words certainly do matter. Replacing the term “virtual particle” with “disturbance” immediately brought clarification for me as it has for many others.

    Given the presence of a particle in quantum field A creates disturbances in all the other quantum fields with which quantum field A interacts do the disturbances in the other fields in turn create a disturbance in quantum field A so A now has a particle and a disturbance? But then the disturbance in field A modifies or creates further disturbances in the other fields, And so it goes back and forth an infinite number of times. However it would seem the magnitude of the disturbances must decrease sufficiently rapidly for the sum of the disturbances in A to be finite. Is this a better choice of words than “an exchange of virtual particles”?

    Many thanks for your efforts.

    Rick

    1. To me, may be I am wrong, nature might have shortcut, instead of continuously disturbing the quantum fields as you said, of converating/compressing (annihilating) the particles and bunch of space particles into zero point element- I call it fuzzy particle, and expanding/uncompressing (creating) other/same sort of particles and space particles. In this scheme disturbance will always be finite.The words ‘exchange of virtual particles’ basically and paradoxically amount to (confirmed) exchange of something (unconfirmed) which we are not sure about. Rest Professor Strassler is the best person to answer in rigorous manner.
      See more details in couple of my posts just above your post.

  55. After pondering over for couple of days I like say that I have gathered some confidence to call my zero point element (as I called it my few posts displayed just above this post) fuzzy particles with following properties:
    1. may be very close to the description of dark matter and Bose-Einstein condensate.
    2. Are compressed by blackholes into very very very tiny particles by combining fuzzy space particles and normal matter particles into one.
    3. May have very very very tiny charge, very very very tiny mass and very very very tiny momentum.
    4. Uncertainty principle and wave-particle duality may allow existence of such particles as planck constants and other limits suggest that there might be certain particles beyond this if nature likes to contract and expand such fuzzy (virtual) particles below and upto planck constants and limits respectively. Then only the dark matter can co-exist with the normal matter.
    5. Expansion of such fuzzy particles at larger scale (large quantities in coherence) is equal to dark energy and contraction of such particles at large scale (large quantities in coherence) are blackholes other wise randomly at smaller scale it appears as vacuum energy.
    6. Since these are fuzzy particles so these can expand into any of elementary particles and space particles depending upon fuzzy (not probability) conditions and may not obey conservation laws below planck constants and limits but above it in expanded state these will. I mean to say nature allows vanishing and reappearance of particles (into and outof fuzzy particles that’s what I guess) alongwith creation and anihiliation of particles.

    Now above scenario can be pure imagination and I might be cheating myself because cheating oneself is very easy than cheating others and according to great scientist Richard Feynman it is next to impossible to cheat (fool) nature and I would like to add we should not involve in mutually assured cheating when we talk about faith and believes in physics like smooth spacetime in GR whereas curvature of spacetime is suggesting that it is not smooth at micro level.

  56. Even particle and field notion perfectly fits that we need to unify particle and field into one which I would like to call zero point element. I shared my views with some eminent physicists. Please find some excerpts below:

    Let me frankly tell you my bad habit first, I apply my logical intuition at first stage in order to solve a problem than gather data in support of it at second stage and lastly introduce necessary mathematics. So please take my emails like that I am at first stage only-so learned persons like you may take my emails as questions rather than answers.

    If we introduce thermodynamics and fluid mechanics at cosmological level then we will logically end up quantization space as discrete particles, and if we take space as discrete particles then next logical question is that we need to introduce (quasi) steady state universe. Bing bang model is logically absurd in its very nature, if balloon (fugga in Marathi) can expand in universe than universe is expanding into what? Is it not expanding from itself and into itself at the same time? If conservation laws are universal and honestly followed is not it the only possibility? If conservation laws are universal than neither universe has beginning nor it has end and not the vice versa? People are introducing/dropping laws of physics at place and time of their choosing and not following them honestly.

    And if we tried to quantize/discretize the space than the space particle must be unified with matter and force particles to arrive at theory of everything.

    Nature is suggesting that vacuum energy and dark matter is nothing but manifestation of all such particles combined into one element I call it zero point element only for my logical convenience as I am yet afraid to call it a particle in normal sense and this zero point element manifests in different ways differently, including spliting into space and matter again, right from absolute temprature to quark soup level temprature, for different time durations. Also due to conservation laws universe assumes the responsibility to maintain definition of absolute temperature and entropy hence contraction and expansion of it occurs. And if I try to imagine further blackholes seems to be like vortex sucking in normal matter and space, and spitting out black matter by compressing both space and matter into zero point element.

    It seems that CMB, accelerating expansion and accelerating contraction is supported by my imagination even I am afraid to call it a thought experiment but this expansion and contraction is intercepted much before zero point contraction and expension into zero point thus always maintaining a steady state because I think CMB is a measure of conversion of black matter into black energy (space) and normal matter, CMB may vary from time to time and place to place and condition to condition and if dark matter has lower and upper limits of clumping together then neither it can form black holes nor Universe containing dark matter can endlessly contract and therefore can endlessly expand.

    Zero point element may have the potential to solve cosmological constant and vacuum energy catastrophe, the discrepancy may be due to that we are over counting the vacuum energy several times whereas it may be just due to zero point element splitting into space and matter temporarily and then recombining space and matter into zero point element again.

    I do not care whether contents of my email are utterly a science fiction but I like to share one thing that it gives me immense peace and joy which nothing in this world can match. And by writing this email to you I am feeling the same peace and joy without knowing the consequences.

    1. Sorry the words ‘dark matter’ and ‘dark energy’ have been written unintentionally as black matter and black energy in my previous email.

  57. I like to request you that these questions can be summed up as matter (the lesser of space), space ( the lesser of matter) and combination of both as Zero point element (which is not like a particle in normal sense and also acts like virtual particles) and this can be answers to many of physical constants, vacuum energy, quantum entanglement, black holes, dark matter and dark energy issues as to arrive at theory of everything we need to unify space and matter and we need to smash big bang theory. Uncertainty principle also points out in this direction that we ultimately need to unify space and matter because at quantum level, as said above, it is in fact lesser of matter (space) and lesser of space (matter) and so we need to modify GR.

  58. Hi Matt 🙂 I have a question about the elementary particles that are force carriers. For example: since the force carrier for the electromagnetic force is the photon, does that mean that the photon always is present where electromagnetism is present? Or that the gluon always is present where the strong nuclear force is present? I know this might seem like a bit of an ignoramus question, but I would really like to understand better the significance of the elementary particles that are force carriers. Thank you in advance!

  59. Beautiful explanation. I suggest virtual particles be named the Strassler Disturbance Particle. 👍

  60. Fantastic article, really helped my understanding!

    Quick question: I read elsewhere that virtual particles that exist for a prolonged amount of time can become real. Is this correct? How is this explained in terms of disturbances? (I am thinking specifically in regard to Hawking radiation)

    Thanks

  61. If you wish for to improve your know-how simply keep visiting this site and
    be updated with the most up-to-date information posted here.

  62. Hi. Thank you for your explanation, it was me much help. Could you tell me what are your references?

  63. Can you help me build a SCEP ? Is it possible to build a Photon Collector ? An Electricity Collector which collects electricity from air ? The Atmosphere ?

  64. I used to be suggested this blog by way of my cousin. I
    am no longer certain whether or not this put up is written through him as no one else recognize such detailed about my difficulty.

    You are incredible! Thank you!

  65. Thanks for a clear explanation!
    Why does a virtual particle that falls into a black hole become a real particle because of the strong gravitational field? If virtual particles are just disturbances?

  66. I also wanted to ask…..If virtual ‘particles’ aren’t bound by the energy mass equivalence equation for example, why do they still need to abide by Conservation of momenta and energy? And isn’t conservation of values such as charge the reason why they form in particle-antiparticle pairs, suggesting the Conservation is in fact necessary even for these weird disturbances?

  67. Thanks for the brilliant article!! It’s been 5 years and so many are still super interested! Fascinating isn’t it, QFT?

  68. Thanks for your thoughts Grigory. Without any math background in the subject, I am struggling to find a good model for the “normal” particle, for as you say, a standing wave isn’t right, since they move; and I’m guessing a soliton isn’t quite right either?

  69. Dear Professor Strassler,

    Many thanks for a very helpful discussion of this phenomenon. As a layman, I have a few questions which I hope you won’t mind answering if you have the time.

    The first one concerns the quantisation of the “virtual particles”. As I understand it, you are saying that unlike normal particles, there isn’t a well-defined answer when you ask, “What is the charge/spin etc. of this virtual particle/generalised disturbance of this field?”. Could you say a bit more about what this means – e.g. does it mean that it simply doesn’t make sense to ask the question? or that the answer is difficult/impossible to calculate or measure? And if it is a calculable or measurable quantity, is it quantised?

    The second question concerns the nature of the disturbances in the field. Am I right in thinking that the difference between a normal particle and a “generalised disturbance” is rather like the difference between a standing wave and an ordinary wave in the sea? And if so, does that imply that a quantum field behaves a bit differently than the sea, with a much higher incidence of standing waves?? If so, is there a simple way to think about why?

    Thanks for any thoughts you (or anyone else) might like to share on this.

    Dominic

    1. Dominic, as another layman i’ll try to share my view on this, let others correct me. A moving normal particle evidently does not look like a standing wave. I personally prefer to think of normal particles (being proper solutions of field equations) as more or less stable and self-sufficient beings that exist on their own. They have no “cause” that causes their existence, while disturbances die out when their “cause” is gone.

  70. In his operate, Trudeau stated that the investigation of Dr.
    This can be as simple as choosing to drink water instead of juice or soda, or fat free milk instead of two percent.
    Fast weight loss which focuses on low calorie intake and fat burning without proper
    nutrition is indeed a starvation practice. Remember, healthy diets are essential for safe and quick
    weight loss. Of program, alcohol consumption to great surplus is the main factor in alcoholic
    liver cirrhosis.

  71. Thank you, that really cleared it up for me. I had thought of virtual particles sort of like ghosts, sort of there but sort of not. This really helped it make sense.

  72. Pingback: Velocity 2X
  73. Sorry not to sound smug, but it seems this whole page just describe how magnet work and how water ripples interact with one another… At least that’s what I understand from this?

  74. This is, by far, the best explanation for virtual particles and how one can think about quantum fields. I have a regular physics meetup with some friends and last week I printed some copies of your post and handed them out to all others. Unsurprisingly everyone liked your post as much as I do, because most books on QFT are very careful when it comes “illustrations” like this. So.. thanks!

  75. Analogy time (and a very close one mathematically); think about a child’s swing.There are outlets to campaign for legal change that are available in the latter that are simply not available (and are even dangerous) in the former.spartagen XT

  76. Dr Strassler: Thank you for continuing to maintain this discourse, I am new to it and appreciate having access to the rich background. I am wondering what happens to a photon, or any high energy particle, once its energy has been expended. If a photon may be considered both a particle and a wave and no longer carries are charge, is it simply a particle? Perhaps a virtual particle (i.e. a disturbance in an existing wave)?

  77. Pingback: Quora
  78. Hey, I liked your article. I’m in 11th grade, so basically I know nothing about sub-atomic particles and my knowledge of Physics and Math is negligible (but present). My questions to you are kinda dumb, but I wanted to know – Are gluons solely virtual? And virtual photon don’t constitute light – so what do they do? And what are virtual electrons? And all this makes me wonder – are actual electrons waves or particles? And what are they composed of?

  79. Analogy time (and a very close one mathematically); think about a child’s swing. If you give it a shove and let it go, it will swing back and forth with a time period that is always the same, no matter how hard was the initial shove you gave it. This is the natural motion of the swing. Now compare that regular, smooth, constant back-and-forth motion to what would happen if you started giving the swing a shove many times during each of its back and forth swings. forskolin natural weightloss product

  80. Hey Great explanations for any layman or a non professional science lover. Hope you will add more info on other topics too

  81. if two electrons pass near each other, as in Figure 1, they will, because of their electric charge, disturb the electromagnetic field, sometimes called the photon field because its ripples are photons. That disturbance, sketched whimsically in green in the figure, is not a photon. buy amazon reviews

  82. Hello Matt,
    I studied chemistry at university (a long time ago) and virtual particles were not mentioned.
    Could you briefly tell me if they are relevant to chemical bonding theory and, if so, how?
    Cheers,
    Robert

  83. Pingback: Quora
  84. Pingback: white wall
  85. Look up the effect of vacuum polarization in high voltage. The virtual electron/positron pairs become “real”. Also if you go way back to the 30’s check out the Dirac Sea conceptualization that P.A.M. Dirac used to postulate the existence of anti-matter.

  86. Dr. Strassler,
    I have two quick follow-up questions to this post:
    1) Are these “virtual particles” quantized? That is – Is the integral over space of the amplitude of these “disturbances” quantized in the same way that it is for a “natural” or “stable” particle oscillation is (i.e. “real” photons are quantized – are “virtual” photons?)

    2) How does this rough, non-“natural” ripple that you describe as being the virtual photon differ from a simple fourier decomposition of a “real” photon field which happens to span a very broad frequency domain in a rather messy way? (to create the impulsive Green’s-type function)

  87. Thanks for the excellent article on virtuals. I was hoping for an answer to a question regarding Casimir’s formation of a virtual particle and whether or not it relates to Coulomb’s Barrier to Fusion. Since virtual particle formation seems to occur in Casimir force, are ‘virtual particle’ transitions a nuclear reaction – albeit a very small one.

  88. Hi,

    Fantastic series of articles, really beginning to shape my view of subatomic particles. I see you have made a fine tradeoff between math (accurate; long time scale of understanding) and plain english (less accurate; but also smaller time scale of understanding) so that laymen like me can understand what you’re saying. I understand better with english but at the same time I’m also burning to see the math when you say things like “special disturbance”, “solution to wave equation” and “greens function associated with this equation”. I would love to read about the mathematical basis of particle physics. I have just a beginners understanding of applied mathematics insofar as a person with masters in engineering can have. Could you recommend a book or two?

    Thanks,
    Satya

  89. Step 2:- Add a firewall exception to the port 25565 (for the purpose of this tutorial)- Add the
    port 25565 to the router under applications, so that the port can be open for you to
    be able to run the server – Step 3:- Run Minecraft_server.
    The essence of the game is to survive by building your own environment
    by utilizing the natural resources and build your
    structures. Once you purchase the server,
    you can install Minecraft Bukkit on it just as if it were your own computer and start playing with your friends right away.

  90. Hello every one i want to use this forum to thank Dr Arigbo for bringing joy into my marriage , i have been trying to get pregnant after my miscarriage 6years ago , but to no avail until i contacted Dr Arigbo, of Arigbo spell temple and Dr Arigbo made a cleanse and pregnancy spell for me , thanks to Dr Arigbo am a proud mother of my baby boy and am expecting another baby very soon , you can contact Dr Arigbo on email (arigbospelltemple@ gmail. com) or call +2347060960158
    for all your spiritual and family issues

  91. hi!,I love your writing very a lot! share we keep up a correspondence extra approximately your post on AOL?
    I require a specialist in this area to resolve my problem.
    Maybe that’s you! Having a look ahead to peer you.

  92. magnificent issues altogether, you simply received a emblem new
    reader. What may you recommend in regards to your submit that you just made some days ago?

    Any certain?

  93. It’s amazing to go to see this web site and reading the views of all colleagues on the topic of
    this post, while I am also eager of getting knowledge.

  94. Hello! Quick question that’s completely off topic.
    Do you know how to make your site mobile friendly?
    My blog looks weird when browsing from my iphone 4.
    I’m trying to find a theme or plugin that might be able to correct this issue.
    If you have any suggestions, please share. Many thanks!

  95. I drop a leave a rresponse when I especially ennjoy a
    poszt on a site or if I have something to contribute to thee conversation. It is a resxult
    of the fire communicated in the post I browsed. And after
    this article Virtual Particles: What are they?

    | Of Particular Significance. I waas actually excited enough too drop a thought 😉 I
    actually do have a couple of questions for you if you tend not to mind.
    Is iit only me or does it seem like some of these comments appear likme ritten by brain dead visitors?
    😛 And, if you aare posting oon additional social sites, I
    would like too follow you. Could you list the complete urls of all your shared sites like your linkedin profile, Facebook pae or twitter feed?

  96. Analysis authors stressed the significance of continuing surveillance of HSV-1 and HSV-2 to screen the
    changing characteristics of the infections and urged
    public doctors to generate more prevention approaches and vaccines against
    herpes infections. Yes, we want one to drive home in a fresh Holden, HSV , or perhaps a used motor vehicle of
    any come up with, but as your premier Hillcrest, QLD Holden dealership
    near Brisbane, we don’t would like that
    to function as last we observe of you.

  97. how do you resist those strong temptations to eat wrong foods and hurt your health and your
    light level. In principle you should always only buy the least processed rice.
    Cool the baking sheet for 5 minutes before adding more cookies to it.

  98. Hello!
    Will everyone hold the exact same issues as i complete?

    I cannot seem to any longer please the partner inside master bedroom.
    Soon after getting 3 young children We’ve shed this baby-belly
    fat nevertheless I cannot manage to re-tighten my personal vagina.

    I can continuously do other activities to make your pet satisfied within the master bedroom due to the fact genital making love no longer excites your pet.
    Help! I’ve began looking at goods and ways to resolve this kind of, may everyone advocate a fantastic one particular?

    Thank you!

  99. When I originally commented I clicked the “Notify me when new comments are added” checkbox and now each time a comment
    is added I get several e-mails with the same comment.
    Is there any way you can remove me from that service?
    Thanks a lot!

  100. Although not all schools in these recent days to continue with a slight decline
    in membership, you definitely know it by impeaching & removing a President
    who usurps power: Federalist Paper No. Your first day there.
    99 percent of it can make an initial Purchase or Balance Transfer.
    It has great passion and a community BBQ.
    Hasselbeck, who is also a tool for checking any prime number up
    to the city and has a wood burning fireplace
    and as a national hero with his physical status, both of the NFL.

  101. The next time I read a blog, I hope that it doesnt disappoint me as much as this one. I mean, I know it was my choice to read, but I essentially thought youd have something fascinating to say. All I hear can be a bunch of whining about something that you could fix should you werent too busy searching for attention.
    retro jordans for sale cheap http://biltfonts.com/images/sys.asp?key=retro-jordans-for-sale-cheap-69

  102. Pingback: Quora
  103. You can purchase this small paperback book used for approximately $2.
    These costs don’t even factor in the purchase of a first car for
    a child, a cost many parents assume or help with when the child turns 16.
    Use extra care when passing in bad weather, at night, or under any other unfavorable conditions, because it is difficult to judge speed and distance.

  104. My programmer is trying too persuade me to move to .net from PHP.
    I have always disliked the idea because of the costs. But he’s trryiong none the
    less. I’ve been using WordPress on numerous websdites for about a yeaqr
    and am concerned about switching to another platform.
    I have heard great things about blogengine.net. Is there a way
    I can transfer all my wkrdpress posts into it? Any
    help would bee really appreciated!

  105. Hi there! This article couldn’t be written aany better!
    Looking through this article reminds mee oof my previous
    roommate! He continually kept preaching about this. I’ll forward this post to him.
    Fairly certain he’s going to hqve a good read. Thans for
    sharing!

  106. Hello There. I found your blog using msn. This is a really well written article.

    I’ll be sure to bookjark it and come back to read morfe of your usseful information. Thanks for the post.
    I’ll certainly return.

  107. certainly like your web site however you have to check the spelling
    on several of your posts. Many of them are rife with spelling issues and I
    in finding it very troublesome to tell the truth nevertheless I’ll definitely come again again.

  108. You actually make it seem so easy with your
    presentation but I find this topic to be really something that I
    think I would never understand. It seems too complicated
    and extremely broad for me. I’m looking forward for your next post, I will try
    to get the hang of it!

  109. Excellent beat ! I wish to apprentice even as you amend your site, how can i subscribe for a blog site?
    The account helped me a applicable deal. I were tiny bit acquainted of this
    your broadcast provided shiny transparent idea

  110. I’m now not positive where you’re getting your information, however great topic.
    I needs to spend a while finding out much more or understanding
    more. Thanks for great information I used to be looking for this information for my
    mission.

  111. I’m amazed, I have to admit. Rarely do I encounter a blog that’s
    equally educative and interesting, and without a doubt, you have hit the nail on the head.
    The problem is something that too few folks are speaking intelligently about.

    Now i’m very happy that I stumbled across this during my search for something regarding this.

  112. Doesn’t the distinction between “real” and “virtual” depend on a fixed background (which of course QFT has, so I’m not disputing anything you say in the article)? It seems to me that, if we were to generalize this to some kind of theory of quantum gravity, you’d need to know which intervals were timelike vs. spacelike to tell virtual particles from real particles, which information isn’t prior to the dynamics.

    I’m an amateur, so I assume some part of what I just said is (at best) wrong. Care to comment, Matt?

  113. Dear Dr.Strassler,
    In the beginning of this article, you use the swing to explain what is “ripple” and what is “disturbance”. The paragraphs around Fig.3 & Fig.4 said that the statement “a particle like an electron is a ripple purely in the electron field” is an APPROXIMATE statement. And the next paragraph you said “The disturbance in the electron field is not an electron particle, and the disturbance in the photon field is not a photon particle.”

    Then my confusion emerged. Before I read the statement “Even to say a particle like an electron is a ripple purely in the electron field is an approximate statement”, I thought as you said that a so-called particle is a ripple in its field and that a so-called virtual particle is a disturbance in its field whatever the field was caused by other particle. So that I confused a statement “The disturbance in the electron field is not an electron particle, and the disturbance in the photon field is not a photon particle”. In that sentence, you said “disturbance” first and then “electron particle”. I knew what you actually mean but confuse the usage of the words. So the correct statement is “The RIPPLE in the electron field is not an electron particle, and the RIPPLE in the photon field is not a photon particle”?

    On the other hand, after I read the black statement under Fig.3 and the so far correct interpretation: “The electron can turn into a virtual photon and a virtual electron, which then turn back into a real electron” beside the Fig.4, does the meaning of ripple remains meaningful? Due to the truth explanation of the electron is the black statement you wrote, we do NOT have to use the word RIPPLE anymore just because it is only the approximation concept? If it would right, then we can get rid of the concept of the ripple? And then said: “the virtual particle is the disturbance of its field, and the real particle is not the ripple of its field but the combination of several, maybe 2, disturbances of their belonging fields”?

    On the other hand, in the article “Particles Are Quanta”, the paragraph under Fig.6 say ” Electrons are quanta in the electron field”. Is that exactly right? Or “Electron is the combination of a virtual quanta(electron) in the electron field and a virtual quanta(photon) in the electromagnetic field”?

    Am I too carp? Or…… 🙁

  114. wonderful points altogether, you simply gained a brand new reader.
    What might you recommend in regards to youjr publish that you
    simply made some days ago? Any sure?

  115. Fantastic beat ! I would like to apprentice while you
    amend your web site, how could i subscribe for a blog site?
    The account helped me a acceptable deal. I had been a little bit acquainted of this your broadcast offered bright
    clear idea

  116. I believe the hypothesis that the virtual particles might be responsible for the missing gravitation called dark matter won’t work. My rationale is that one would expect the gravitational force due to virtual particles to be nearly uniformly distributed (homogeneous) whereas the few calculations that have been made of dark matter distribution are not consistent with this.

  117. Hello, Neat post. There’s a problem along with your web site in web explorer,
    may check this? IE still is the marketplace leader and a big part of other
    people will omit your excellent writing due to this problem.

  118. Dear Matt,
    excellent article. Maybe it is a rather technical question but I try: what is your idea on what is often called “pomeron” ?
    Is it a very peculiar disturbance of the gluon field?
    Thanks.

  119. In my seldom humble opinion, it makes no sense to talk of real particles as real and virtual particles as a mistaken terminology. The reason I say this is because all we can ever have in physics are models that are hopefully predictive. The idea is of a particle itself is a model dependent concept based on our classically wired brains. So in my opinion, virtual particles are as real , or as unreal , as so called “real” particles. I think as models go , virtual particles are a very useful and visual way to model things in QFT.That’s why this terminology has persisted despite the criticisms of more purist thinkers.

  120. I hopee the testimonial weight loss group names youu have just a
    few hundred dollrs too free. Hypnosjs iss onee way to reduce their body size.
    Nutritional deficiencies after the surgery to ensure that
    you expand your current workload. If you are looking pro a splendid
    quick consequence loss ways with thhe intention of losing weight method.
    While a recent weight loss group names study, supplementing a
    healthy diet.

  121. Hey Professor Strassler,
    Great article. Right at the beginning what caught my eye is your comment about how you were once a “layperson” yourself, at age 16, reading these types of articles. i should say I am in that particular situation; I am 16 as well, trying to gobble up as much scientific information about physics in general as I can. Do you have any recommendations on where to go and what to do during my high school years so I can prepare myself? And is there anything that can be done to get a “feel for” the math involved in this (unfortunately I don’t think I will be taking differential equations until I reach University).
    I think it is great to be interested in the fields of physics (pun intended) and thanks a lot for putting this website up, it is very informative and useful.
    Thank you!

  122. Great article thanks. Learned a lot.

    Small typo, repetition of “in”

    electrons spend some of their time as a combination of two disturbances, one **in in** the electron field and one in the electromagnetic field.

  123. Hello prof s.
    can you weave your magic to explain gravitons, mass, gravity, and how gravity works at huge distances – ripples in the gravity field?

  124. sir, i am not a physicist so i want to ask you that many books say that the so called virtual particles have more energy than that of its initial condition so they don’t conserve energy will you please clarify that they don’t conserve energy thanks

  125. Dr. Strassler,

    Perhaps you can clarify for me a point (about black hole “evaporation”) that I can’t seem to get straight. I don’t understand the “energy arithmetic” involved. Here’s my puzzlement.
    If a virtual pair is created just outside the event horizon, and one of the pair goes in and the other out, it looks to a distant observer that a particle has been emitted. Fine. But the other one has gone in and *added* its mass (or energy, same thing) to that of the black hole. How does the net mass/energy of the black hole decrease?
    And if one would argue that the entering particle is an antiparticle half the time (fair enough) and would annihilate with an existing particle inside, that only eliminates the mass, not the net energy inside. And even if one says that the virtual particle pair is created slightly inside the event horizon and one of ’em tunnels out, that’s fine, too; but still: how would this decrease the mass or energy of the black hole?
    It is interesting to me that if radiation from a black hole *is* due to virtual particle creation near the event horizon, then the rate of evaporation would seem (to me) to be proportional to the surface area of the event horizon, which is proportional to the square of its radius, which in turn is proportional to its mass. So larger holes oughta evaporate faster (which apparently *isn’t* true) unless the severe bending of spacetime in this neighborhood affects these admittedly Euclidean calculations.

    Help, please! Thanks!

  126. For those of you who have made your necessary checks and would like to get the app, it can be acquired at this Download Link for $4. Full nutritional and allergy information on chili and all other Wendys menu items can be found at.

  127. Hey this is somewhat of off topic but I was wondering if blogs use WYSIWYG editors or if
    you have to manually code with HTML. I’m starting a blog soon but have no coding experience so I wanted to get advice
    from someone with experience. Any help would be enormously appreciated!

  128. Dear Matt Strassler:

    Suppose you try to make a distinction between the realm of concepts and the realm of objects, so that with virtual particles it is all in the realm of concepts, and we humans are still in need of locating the objects to which they the virtual particles correspond to in the realm of objective reality.

    Can you explain virtual particles in that manner?

    Or it is all impossible because of the complexity of the issue whatever, and without a complex and complicated mind, it is impossible from your part to explain to them folks with no such complex and complicated mind.

    Marius de Jess

  129. Reading your articule has improved my understanding greatly.
    Could you answer a couple of short question that appear not to have been raised yet.
    1) How does a real particule create a disturbance in the field surrounding it? Is the disturbance a superposition of the real particules wave function and the background virtual disturbances caused by the uncertainity principle. As I understand it the disturbance can extend to a quite large distances although at large distances it is also very weak.
    2) How do these disturbances cause an attraction and repulsion of electrons and positrons. Ie through the neutral photon field.

    1. It all has to do with the interaction of one field with another (or even with itself, in some cases.)

      To answer these questions actually requires solving the equations that describe the interaction between the electron field and the electromagnetic (i.e. photon) field. It’s similar to solving the equations for the electric field due a charged particle in ordinary first-year physics, but including quantum mechanical effects and the fact that the charged particle is itself a ripple in its own field.

      But anyway, at this point you actually have to solve equations… not so difficult ones, but too advanced for this website.

  130. I think we need to re-evaluate what we mean by “real” and “virtual”. Every physical phenomenon is “real” in the sense that it can be observed (whether indirectly or directly). I’m not implying any assumptions on anybody’s understanding (surely, my own is quite lacking), but giving up preconceived notions of what “real matter” is (as opposed to unreal matter?) is indispensable in coming to grips with modern physics — no need to get metaphysical either. Of course, there are some deep metaphysical questions which are highly relevant to human interests into what IS “reality”. Virtual particles are some kind of fluctuation in a field, so are real particles… they are just two different kinds of fluctuations. That’s it, right? The pictures above where the electrons are the particles and “interact” by “exchanging virtual particles” is misleading as well (it’s a model, after all) because the electron is also simply a special type of disturbance in the electron field. The real particles are not even “real”, in the sense that they are not actually “solid globs of stuff”. The field IS the underlying “real stuff” — similar to the collection of water molecules being the real stuff and ice cubes aren’t really “real things” in the sense that they are just collections of water molecules. The water molecules are the actual real things. The ice cubes are just a particular arrangement of water molecules.

  131. Hello professor Matt Strassler,

    Just a second pair of questions.

    As I wrote in my previous post: it helped me a lot by getting rid of the idea that a virtual particle is a kind of real particle. Maybe too many textbooks suggest that a virtual particle is a kind of very short lived “real” particle which just can’t be observed because it disappears very very fast. But still is real.

    Right now I am reading “Deep Down Things. The Breathtaking Beauty of Particle Physics” by Bruce A. Schumm (I would recommend this book). It’s maybe trivial, but just realizing that W bosons (mediators of the weak force) are neither called leptons or are made up of quarks shows that W bosons are not real matter. Right?

    I am still confused why a mass can be determined for W bosons. How does this fit in your explanation above? I also wonder how the Heisenberg uncertainty principle fits in your explanation.

    Best wishes,

    Richard
    The Netherlands

  132. Hello,

    Great explanation. It greatly helped by just stating that one should just forget the term “particle” when talking about virtual particles.

    Isn’t it possible to make this explanation possible as a pdf-document (easier printing)?

  133. Great post, as usual.
    There is however an important point that you did not tackle: what is the relation with the uncertainty principle? (every book mention it without actually explaining it much…)
    I think it’d be good if you address this issue in a revised version of this post (if you ever revise it).

  134. Thank you for the post and responses to the many good questions of your readers. I have learned a lot from both. Please keep up this effort and get your book published soon so we can all read it.

  135. Hi Professor Strassler,

    I read through the article and the comments. But I’m not very good at science so I’m not entirely sure if I understand the explanation. I was wondering if you would be willing to tell me if I got it right per below:

    Some experiments showed that when two electrons pass by each other in a vacuum, they are repelled as we would expect (negative charge + negative charge = repulsion). But at that moment, when the electromagnetic fields of the two electrons interact, they create a disturbance. This disturbace is short-lived and has a variety of properties such as mass, momentum, energy, etc. The disturbance could be loosely described as a “shadow” of an electron. This is called a “virtual particle”.

    Likewise, in a similar manner, when a positron and an electron pass each other, their positive and negative fields interact and create a disturbance. This disturbance is called a virtual photon. But like the virtual particle, it’s not an actual, “real” particle (like an electron).

    In other words, the virtual particle really is a popularized term that describes all manners of these disturbances between real particles. There are various ways to calculate the disturbances and one popular method is called the Feynman Diagram. The lines in the diagram are called “virtual particles”. In other words, the disturbances exist but technically speaking, virtual particles only exist in a Feynman diagram. We can’t even call the disturbances “virtual particles” because depending on the method of the calculation, the term “virtual particle” might not even be a part of the language used in the particular method. Technically speaking, the term “virtual particle” is a convience, a way of describing one part of a mathematical diagram.

    Mathematically speaking, it can be shown that with enough energy, the disturbance (or “virtual particle”) can actually result in a real particle. So instead of a disturbance/shadow of a real particle, a real particle can actually be formed from the disturbance.

    Also, particles such as electrons are really just long-term, stable disturbances in certain fields. For example, an electron isn’t a little ball found in textbooks – it’s a disturbance in an electron field. The disturbance in the electron field is called an electron.

    If I got it right, I was wondering about a few things related to cosmology:
    1. Does String Theory describe the factors/events that cause these disturbances (the “virtual particles”)?
    2. Is it possible that the Big Bang came from one very energetic disturbance that created real particles in a similar manner to how “virtual particles” are created but on a much larger scale? Would I be way off mark to say that the interaction of the fields in an “other” state (prior to the universe’s existence), disturbed enough to create an event similar to a virtual particle? (The event being the Big Bang.)

    Hoping you can shed some light!

    1. You’re pretty close on most things… a few little details are wrong, but your general impressions are in pretty good shape.

      1) The “factors/events” that cause these disturbances are already described by quantum field theory. String theory doesn’t really change the story I’ve told you. There are “virtual strings” too, which are disturbances in string fields.

      2) Yes, perhaps. It’s actually much more subtle than you suggest — but the suggestion that the universe emerges as a disturbance in space-time itself has certainly been made.

      1. Thanks for the reply! I was wondering if you could correct/point out where I got it wrong. I probably won’t understand it all if it’s too technical but I’d love to have a starting point to learn.

        I don’t think I see any articles on quantum field theory in your collection. I was wondering if you might be able to write one? Or maybe provide a dumb-down version of it?

        What would you say is the most common thought on how the universe emerged among scientists like yourself?

  136. Thank you for writing this blog.

    I think sometimes virtual particles are taken too serously, as if they “really exist”. The idea of virtual particles is advocated strongly and many people (especially laymen) think they are therefore “real” or at least have “real observable effects”. But the fact is they appear only when we do calculations using perturbation theory. In this sense I would say that they are due to the perturbative approach and are nothing more than a mathematical tool, which brings intuition and helps to visualize (through Feynman diagrams) which contributions are important.

    The quantum fields are the fundamental objects and if we could perform all calculations without perturbation theory, there would be no virtual particles. The properties we calculate using perturbation theory and virtual particles are not due to virtual particles; we simply use this approximate method to calculate these effects of the fields.

    There are also cases where perturbative approaches do not work (thus virtual particles stop existing?). Further, as is pointed out, for example, in this paper*, virtual particles can also be utilized in classical physics. Yet, not many physicists say that classical phenomena occur due to virtual particles.

    * http://arxiv.org/abs/quant-ph/0609163

    1. Of course I agree with you on all the technical points.

      Pedagogically, there’s an issue. “Virtual particles” have entered the public lexicon; if you’re going to explain things, you have to use existing language.

      My choice is to treat “Virtual particles” more generally than simply lines in a Feynman graph — to describe them more generally as “what fields do when they aren’t rippling as real quanta”. [Solitons or instantons would not fit in this category.] I find this covers most situations — in particle physics, what is most commonly encountered pedagogically is Green functions with some kind of boundary condition (this includes the classical phenomena and the lines in Feynman graphs) and integrals over Green functions (loops in Feynman graphs) or resummations of sets of lines and loops (as happens when a quark turns into a jet of quarks, antiquarks and gluons.) This way of talking gets away from the verbiage of treating “Virtual Particles” as though they have a reality beyond the math, and treats them not as “particles” but as “field phenomena”. It doesn’t quite get to the point of saying that they are calculational artifacts… but that’s because the field phenomena which they represent are presumably *not* calculational artifacts… yes? In other words, I feel it is useful to describe “Virtual particles” in terms of the field phenomena that they are used to describe… and so far, it has worked very well on this website, in that it hasn’t caused many pedagogical contradictions.

  137. A really quick question. The disturbance in the field that pushes two electrons apart, is that a classical electro-magnetic field? So you have one quantum electro-magnetic field; and, where there are disturbances in that field, you have classical electro-magnetic fields – of the sort created by say a charged object or a magnet.

    1. A classical electromagnetic field between two electric charges is a very special “disturbance”. However, most disturbances are not classical electromagnetic fields.

      1. Dear Dr Strassler,

        I realize you havent commented in over three years but I just wanted one clarification after ive read through your comments.

        The concept of these “disturbances” i.e. virtual particles being separate from the concept used in Feynman diagrams was something you agreed with .

        Why shouldn’t Feynman diagrams interact via these physically real “disturbances”. You even have a Feynman diagram in figure 6. Are you meaning to tell us the interpretation you’ve presented in this article is not the virtual particles in Feynman diagrams? then what is the purpose of this post?

        I realize youve got a lot of questions in the comments but i can assure you my gratitude and insistence exceeds that of the other commentators

  138. First of all, thanks for keeping this blog! I only discovered it a couple weeks ago. I first came across your article about the structure of the proton and it has changed my view of the world. The picture of 3 quarks and 3 gluons just never sat right with me. Thinking of the proton as a “bound state”, essentially a cloud of a very large number of particles buzzing around, but with a particular net balance of them makes so much more sense to me and was a groundbreaking revelation!

    So if I understand this virtual-particle article somewhat correctly:

    — Virtual particles are not particles because they are non-resonant, i.e. the properties of the virtual-particles like energy, wave amplitude and frequency are not at the limited number of specific values (quanta) as that of the resonant particles. In other words virtual particles are not particles because they are not quantized. That’s all the term particle means in quantum mechanics, is that the phenomenon is quantized. Does this make some sense? So virtual particles are not quantized? In other words, they can have any arbitrarily small (or large?) energy, amplitude and frequency, etc.?

    — Maybe I am misunderstanding, but there seems to be a link between the uncertainty principle and virtual particles. For example, in the “universe from nothing” question, the “nothing” is actually a null quantum field that is buzzing with the life of virtual particles. It’s nothing in that there is nothing to observe until a “real/actual” particle comes into existence. The virtual particles cannot be measured because they fall within the limits of uncertainty. So in order for a real particle to emerge from virtual particles, it seems like many little virtual-particle-ripples have to coalesce until their quantifiable properties go above the limits of uncertainty, then a quantized amount of that energy might fly off as a result and “bang” we have a “real” particle… i.e. a quantized field fluctuation which is somewhat long-lived.

    I am envisioning virtual particles like the random and chaotic ripples on the sea, and real particles like the waves that emerge and propagate to the shore. All real particles decay into “nothingness” just like the waves break and crash, but there are always a seemingly unlimited number of virtual particles of all shapes and sizes (i.e. of randomly varying magnitudes and frequencies of ripples on the surface of the sea, etc.). Is this a decent analogy?

    If I am sort of on the right track, then this brings up the question: If two distinct quantum fields have a virtual particle that has the same quantifiable properties, are they different? If the electron field has random fluctuation X and Higgs field has random fluctuation X (all “virtually-measurable” quantities are identical and encapsulated in X), then what makes those virtual particles different? Or are they? In other words, is it a necessity that there are different fields? Why can’t there just be a single field that gives rise to all the particles, and the particles simply vary by their quantifiable properties like amplitude, energy, etc.? Of course, this begs the question on the existence of the fields and whether or not they are simply mathematical abstractions (not real) used to explain the existence of observable (real) phenomena.

  139. Dear Prof Strassler,

    Thank you for your excellent article explaining virtual particles. Your article was eminently readable and immensely informative. I learned a lot.

    If you are still entertaining to questions on this topic, I do have a few.

    First, is there now a consensus among professional physicists as to the ontic nature of quantum fields? Many undergraduate chemistry textbooks still seem to suggest that the quantum field for the electron is just a mathematical construction. They say the wave equation for the electron has no physical interpretation and it is only the square of the wave equation that has meaning, that being the probability of finding an electron in a certain location. I was surprised and pleased to read in your article that quantum fields are to be understood as real entities, in and of themselves – not only real, but fundamental. I had already come to the same conclusion myself, but I had never seen it so plainly stated as in your article. My reasoning was the same as yours – a field can exist without a particle, but a particle cannot exist without the field. So how accepted is this interpretation among physicists and has anyone told the chemists?

    Along these same lines, I wonder if could you say something about the distinction among the following fields which are easily confused:
    1. Electric Field, as between two charged particles or between plates of a capacitor.
    2. Magnetic Field, as between two poles of a permanent magnet.
    3. Electromagnetic field.
    4. Electron field
    5. Photon field
    6. Ether.

    With my new insights gleaned from your article, I would like to take a stab at answering this one. Please correct me where I am wrong. As I understand it, fields 3, 4, and 6 are really the same thing, that is, the medium for the propagation of light waves, but that it is taboo to use the term “ether” for historical reasons. I like that you call it the photon field. That seems to me to be the most unambiguous way to put it. I had not seen it called that before. It is the quantum field for the photon.

    Field 4, of course, is simply the quantum field for the electron which you clearly explained in your article. The important point here is that the electron field is not the same thing as the electric field nor the electromagnetic field.

    Fields 1 and 2, strictly speaking, are not quantum fields but rather a mapping of a physical property to points in space as one might also have a temperature field or a pressure field or in solution chemistry one might have a concentration field. As such, these fields do not transport energy by wave action, but may represent a kind of conduction or flow of some sort down a gradient. What is it that is flowing in the case of the electric field and the magnetic field? Textbooks call these, electric flux and magnetic flux, but again suggest that these are just mathematical constructs and that nothing is actually flowing. Would you say there is a flow of virtual particles or disturbances in the underlying electromagnetic field? Also, is the magnetic field completely reduceable to an electric field, perhaps a circulating electric field? Does a magnetic field boil down to a particular constellation of circulating virtual particles?

    I hope you can further educate me on these topics. I tutor undergraduate students in introductory physics and chemistry. While they are not required to know these details, a deeper understanding on my part helps me teach them what they do need to know.

    Thanks.

    1. Oops, I see some typos in my post. Most importantly, the phrase “fields 3, 4, and 6 are really the same thing” should read “fields 3, 5, and 6 are really the same thing”. Less importantly, in paragraph 2, the phrase “If you are still entertaining to questions” should read “If you are still entertaining questions”. Sorry about that. -Steve

  140. So could a virtual particle pair | disturbance in the electron field have transient and very highly localised effects on spacetime? If so, that would – when taken cumulatively – result in quite a large effect on the universal scale. Say, enough to account for the effects of dark energy?

    1. my thoughts exactly, I done a search to find anyone else with my thoughts on this and you are the only one.

      For some time now I have postulated the idea that these ‘virtual’ disturbances, brief as they may be, are enough of a force to leave a residual gravitational effect. As you say cummulatively enough (maybe, as I do not have enough information to calculate) to account for dark matter – we have to have gone dramatically wrong somewhere to be looking for invisible matter – doesn’t feel right, where as this, well maybe!!

  141. Gee, declaring virtual particles unreal by virtue of the concept of time, which is itself “virtual”.
    that is clearly non sequitur. “Selective science jiggerying”, eh?

    yours truly,
    virt-u-oh-so

  142. Come to think of it: In the early universe the gammas may not represent photons but some other bosons, maybe gravitons. And general relativity is non-linear, where energy can be converted into mass(at enormous temperatures total particle energy is mostly kinetic), and so gravitons can beget particles(electrons and positrons) which then ultimately meet and decay electromagnetically to photons.
    Does it now make sense?

  143. Henrik
    In the symbolic equation I had a two way arrow which was swallowed when I posted it

  144. Dear Mr. Strassler
    If a photon meets a photon they may sire a particle-antiparticle pair of some kind – if their energy is high enough. But photons do not interact accoding to the linear electromagnetic theory. Is it a second order process then in QFT, “pro- ducing” virtual pairs?
    In the (very) early universe for a brief, brief moment, I understand, there was equilibrium between radiation and matter before expansion cooled it to a one way street leaving precious little matter and a quite bunch of entropy in the form of photons. But what are these very very early interacting photons and how do they do it? Virtual second order proces-ses?
    It all comes from me contemplating the symbolic equation : gamma + gamma electron + positrion (or some other pair of particles) which I remember having seen in porpular science books or met on the net.
    Does it make sense?

    Yours sincerely
    Henrik

  145. As a nascent grad student in theoretical particle physics most of this information isn’t new for me, but it is always great to hear another person explain it. Thank you for the clear and interesting explanation!

  146. well its pretty intreseting but i want know why do we need to introduce the concept of virtual particle

    1. Quantum field theory requires there be disturbances of this type, and quantum field theory agrees with data. So we have to have these disturbances.

      Why call them “virtual particles”? Because it proved useful for certain types of calculations to organize the disturbances mathematically as though they were made from a sum of many fictional particles. [If you know some math, this has to do with the usefulness of Fourier transforms.] But that’s a math issue, not a physics issue. Unfortunately, in hindsight, this math fact got taken a little too seriously… its limitations are much better understood today than was the case when the notion was introduced.

  147. Dear Prof,

    Since you are the foremost authority in virtual particles with superb mathematical skills, I wonder if you would be kind enough to point out to me what is wrong with the equations below because the result does not make sense. I am sure there is an error somewhere.

    NOTHING CAN TRAVEL FASTER THAN LIGHT IN AN ABSOLUTE VACUUM BUT:

    sqrt = square root

    10 = 3.1623^2

    i = imaginary number = sqrt (-1)

    i ^ 4 = +1

    *****

    E = mc ^2 {(1-v^2/c^2)}^(-1/2)

    If an imaginary particle is travelling at 3.1623 x C,

    E = mc ^2 [{(1-(3.1623c)^2)} / c^2]^(-1/2)

    E = mc ^2 {(1-10)} ^ (-1/2)

    …. = mc ^2 / square root (- 9)

    …. = mc ^2 / 3i

    If we raise E to the power of 4,

    E^4 = (mc ^2)^4 / 81i^4

    Since i^4 = +1,

    E = 1/3 x mc^2 when an imaginary particle is travelling 3.1623 times the speed of light in a vacuum ! ! !

    THIS APPEARS TO MAKE NO SENSE.

    OR TO PUT IT IN ANOTHER WAY, IF A PARTICLE MOVES FASTER THAN THE SPEED OF LIGHT, IT’S ENERGY DECREASES AND IF IT’S SPEED REACHES INFINITY, IT’S ENERGY AND HENCE IT’S MASS BECOMES ZERO !…….. unless it’s energy is compensated by it’s increase in it’s Newtonian Kinetic Energy (which may not work either as the mass becomes zero, the Newtonian kinetic energy will also become zero);

    E = mc^2 + 1/2 x mv^2,

    The only time when matter moves “faster (though not strictly true)” than light is during the period of inflation at the time of the Big Bang.

    At this time the total energy and mass of the universe may be small (and not infinitely large) but as the inflation decreases towards the speed of light C, the total energy and hence the total mass may be at it’s maximum at C.

    From then on decreases exponentially to the total energy and mass of today i.e. at rest of E = mc^2.

    To put it in another “nonsensical” way, it appears that the Big Bang at the beginning of time comes from a singularity that do not have an infinite density but rather a singularity with almost zero mass but with infinite velocity!

    Thanking you,

    Warmest regards,

    Dr Looi

    email: looihw88@gmail.com

  148. So virtual “particles” are short-lived disturbances which occur within fields. Is that a correct enough summary?

    If so, I have another question in regards to the Casimir Effect. I understand that the experiment is run with super-thin plates which have all electrical charge removed, in a vacuum. A few places online, I have seen the claim that the virtual “particles,” the effects of which are observable in the experiment, are literally generated from nothing. But if virtual “particles” are short-lived disturbances which occur within fields, then it seems they are making the claim that a disturbance in the field can occur without a field. What am I to make of this?

    1. Zia, I would say a quantum field is a mathematical concept for each point in Space. The field is not a real structure but the virtual particles popping in and out of existence are. The reason the virtual particles can come from nothing is because, for a very short time, they can borrow energy from the universe as long as they give that energy back very quickly, which they do, due to pair annihilation. If i am wrong then please, Professor, correct me !

      1. Thanks, but I’m having a hard time seeing how “energy borrowed/obtained from the universe” is “nothing.”

        1. These particles are not really coming from “nothing” as the universe they are borrowing energy from is not nothing at all. However, if the universe didn’t exist, would virtual particles still be able to “borrow energy”? I mean start with the initial state of there being no particles whether real or virtual… would virtual particles still blink in and out?

          I’ll add my $0.02 here, as I’ve thought about this some — please go easy on me as I’m somewhat of a mathematician and only read physics for personal interest.

          It’s not that the field is “nothing” in some philosophical sense, like it is absolute void (no space, no time, no things). The scientific theory doesn’t really say what a field is or isn’t (in terms of some ontological reality). What the theory says, is that “this particular mathematical equation describes our observations of ‘things'”. People will argue endlessly on what ‘nothing’ means. But that is a question for philosophers, the science is simply meant to take in observations and predict future observations… observations of “things” (electrons, photons, etc.).

          It is a highly significant fact that the equations that describe our observations predict that particles pop out of “nothing”, but it doesn’t prove that in a “true void of nothingness” this would happen, as that entails the assumption that there would have to be no observers around to make the observation and confirm that the observation fits with the equations. So we just fall into a trap in not doing science by attempting to answer that question, but discussing philosophy instead. However, I still think it is an important endeavor to do both.

          In terms of a scientific realist perspective, an equation isn’t a “thing” at all… the ink particles it is written in or the electrons representing it as digital bits are real observable physical things. In other words, equations don’t have any “real” existence to them. “Real physical existence” is limited to those physical things that we can observe. Of course, that begs all sorts of other questions, and scientists will argue endlessly on what “physical things” actually “exist”. There’s a bit of circular logic going on here in all perspectives, really… something exists because we observe it and we observe it because it exists.

          I have satisfied myself somewhat on these issues by clarifying what science actually is, and that is, making observations and formulating models/theories/etc in order to predict future observations. That is science. Any discussion of what “things actually are” is philosophy, not science. Of course, scientists should be free to discuss philosophy openly, however, one may risk bogging down the progress of their scientific work by getting sidetracked too much by the philosophy — of course the opposite can happen too, that the philosophy can spawn new insights and contribute to new scientific theories.

          An obvious question hanging over all this is: What does it mean to observe a thing? Does observing an electron count as observing the electron field and prove the existence of the field? Does observing patterns of clicks in a machine count as observing the particles themselves (in a philosophical sense, not in a scientific-theoretic sense)? Other than DIRECT observation with the human senses, it gets philosophically jumbled somewhat. Personally, I tend to side with the perspective that “no thing actually exists” — in the sense that if a field is actually nothing (not really a solid physical thing in the sense that we think of solidity and nothingness as being distinct concepts), then every thing that is derived from that field of nothingness is actually still nothing as well (emptiness, void, not actually solid in the sense that it isn’t distinct from empty space/void/nothingness). That physical reality is only observation/perception is the inevitable conclusion. Of course, that begs a whole host of philosophical musings. Either way, it does not diminish the success of the scientific method at predicting future observations from past observations.

          Ok, end rant, sorry if this is seen way off topic, but these questions seem relevant and natural to the topic of virtual particles. I’d be really surprised if there are any physicists out there who do not ponder these things and have their own opinions!

  149. This is great stuff! I actually came here because of trying to understand something about classical fields and got sidetracked into virtual particles.

    Something I was wondering:

    (slight diversion) You know the classic situation of a current going through a wire, with a static magnetic field around it that radiates out a certain distance? People used to say some bizarre things to me about current putting it’s energy into sustaining that magnetic field when accelerated, and then drawing it out again to resist deceleration, and that that was basically what impedance was. At the time it seemed bizarre that the current should have the energy I was told about, and then have all this extra energy as well in the magnetic field. To explain this, I assumed that energy transmitted by the current was both the energy in the electrical motion and in the associated magnetic field, which combined added up to the normal momentum and energy transfer I was told about.

    Are the normal virtual particles basically the same thing? Where we say an electron has these properties, but any real electron’s properties are also given by how it “blurs” into these other fields, existing maybe as a comparatively more localised electron and a halo of these other interactions? If so, do you have to change loads of things about the properties of a “naked” electron to get the same thing when you add in all the effects of the fields it’s interacting with?

    1. I’m having a little trouble understanding your question. This is probably partly the way you’ve phrased it and partly that it has been ten years since I taught freshman physics on impedance etc., so I’m probably forgetting the cause of the common confusion that I think you’re expressing.

      Classical fields are related to certain aspects of “virtual particles”. But virtual particles represent a much more general class of phenomena.

      How energy is stored in currents of moving electrons and in the fields that they carry with them is a tricky business. But I’m not sure exactly what I could say that would clarify your question.

      But I can answer this. Yes, a “naked” electron is quite different from the real, physical electron. In fact it’s not unique what “naked” means because you can’t actually ever strip away all the junk that the electron carries with it, so it’s really just a figment of our imaginations to think of it naked. However, there are different classes of things inside that halo, and some of them fall away with distance much faster than the classical electric field, which is the thing that drops away slowest with distance [like 1/(distance)^2]. That’s why in most of undergraduate physics, we only have to deal with the real electron and its electric field; all the complexities of the real electron are typically important only for measurements which take place at subatomic distances.

      I hope that helps a little.

  150. Hi Professor, I am hoping you could help me understand what quantum fields are. As i understand it, the Electromagnetic field purveys the whole of space but what is the field made of ?

    Thanks

    1. As far as we know today (though you must remember that in the future we might learn more about the universe that would change this viewpoint), everything in the universe is made from fields. The fields are the fundamental ingredients in the universe, according to current views. They aren’t made from anything else. Everything else is made from them, and their ripples.

      1. Hello, along a similar line of Kevin’s question, where do the fields “come” from? Or I guess originate might be a better choice of words. Also, I’m curious, if I understand your explanation of virtual particles, then what causes these fields to “interact” or “disturb” when there aren’t any particles around? I think I read somewhere above in the comments where someone used the waves of the ocean as an example, but even then, are not the waves of the ocean caused by a variety of factors?

  151. Hi Matt, thanks for your explanations, they really help people like me who are interested in the concepts of physics but are useless at maths.

    I think i have a simple understanding now of what virtual particles are. Here is my explanation..

    So a photon is a ripple or excitation in the electromagnetic field or an electron is a ripple in the electron field. I like the word ripple so we use that from now on ! When these particles travel through there respective fields they can interact with other fields causing disturbances in them. These disturbances are what we call virtual particles.

    But also, when there are no particles, the fields interact with each other causing disturbances which we call virtual particles. The difference is, a normal particle is a ripple in the field that can travel through it. But a virtual particle is not, it is a disturbance in the field which is a totally different thing.

  152. I think you should consider a full fledged book with all the required math and your exceptional interpretations of equations in terms of physics.

    It is probably too much to clarify each and everyone’s confusion without the necessary math. In the language of the Chinese proverb , teach us to fish rather than providing a piece meal.

  153. Professor, please tell me if this makes sense to you:

    Even on its own internal logic, Feynman diagrams do not imply the real existence of virtual particles. This is because each virtual particle is associated with with a single line in a Feynman diagram but we do not get an end calculation of anything except by summing over diagrams, (or “histories”). I am not familiar with the calculation itself but I suspect that the intermediate values associated with the virtual particles cancel out in the end and do you even get a specific numerical result but rather a probability distribution. Feynman diagrams work only because the lines correspond to, (probabilistically), real component waves, (disturbances), in a quantum field. You are actually just using the diagrams as a tool for deciding on terms to include in a perturbation calculation. The virtual particles then are just mathematical fiction.

  154. Dear Professor Strassler, Tim Martin asked: Also, why do you answer some people’s questions, but not other’s?.
    You responded: do you really want me to answer that?
    Tim said: But also, I have questions I’d like to ask on other posts of yours, as well, and if certain questions are more likely to get answered than others, I’d sincerely like to know what those criteria are!

    Like Tim, I have also noticed that some (reasonable?) questions don’t get answered. I exclude the questions from obvious trolls, or arguments on topics outside of physics. Some kinds of questions *are* more likely to be answered than others, and we aren’t sure that if we ask a question, it will be answered. So, we would like to know:

    – which kinds of questions are likely to be answered?
    – which kinds of questions are NOT likely to be answered?

    We do understand that you retain the right to decide whether or not to answer any question; all we are looking for is some guidelines.

    Regards,
    pdcjw

  155. Hi Professor,

    Your article, and the comments are especially helpful. I am still confused however, and I think that this confusion has to do with my attempts at understanding other descriptions of what virtual particles are. Prior to all of this I was under the impression that virtual particles are particle pairs that instantly pop and in and out of existence in empty space. And I though that they were also called quantum fluctuations, or quantum energy. But when I read your description, I am imagining them as ripples in this quantum foam. Stable ripples are what we understand as particles and the unstable ripples are known as virtual particles.

    What is the difference between a particle and a ripple or disturbance in the foam? When people say “particle” I think of an atom, or a subatomic particle. And this cosmic foam, is it the Higgs field? From my understanding, it is the Higgs field that gives particles their mass based on their movement through the field. If you could clarify these points for me that would be much appreciated. Thank you.

    1. A particle is a ripple in a quantum field. I never used the term quantum foam, I believe. Nor is anyone’s use of the term “foam” to be confused with the Higgs field; there is no relation whatsoever. Let’s back up.

      There are fields in nature. These fluctuate randomly — these quantum fluctuations are often described as virtual particle pairs, but this is not entirely accurate which is why I avoid doing so explicitly. In addition to these random fluctuations, the fields can also have nicely behaved waves on top of those fluctuations. That’s what I call a ripple. The ripple with smallest allowed height is called a “quantum”, and also called a “particle” in modern parlance. If you think of particles as little dots, like dust specks, then you are not understanding what the modern conception of a particle is. A particle is a quantum — a nicely behaved ripple in a quantum field.

      More general disturbances in quantum fields include the things we call “virtual particles”, but those things really are not particles at all. To understand all the different things that quantum fields can do, you really need a course on the subject.

      It is not correct that the Higgs field gives particles their mass based on their *movement*. Many people use analogies that suggest this, but the analogies are inaccurate. The Higgs field gives mass to particles even when they are standing still.

      I have explained much of this at a level appropriate for people with a little math background in a sequence of articles, http://profmattstrassler.com/articles-and-posts/particle-physics-basics/fields-and-their-particles-with-math/ and http://profmattstrassler.com/articles-and-posts/the-higgs-particle/how-the-higgs-field-works/ . If these are too advanced, stay tuned; I will eventually write less advanced articles on the same subjects… but that’s not easy.

  156. Hi Fred: I am trying to reconcile your answer with the one given by Matt, “A photon traveling from the Pleiades is clearly about as close as you are going to get to the ideal; its energy and momentum are almost the perfect match that you would expect for a massless particle.” Should your definition be rephrased as “virtual” means significantly far from being “on mass shell” and real/non-virtual means insignificantly far from being “on mass shell”, because nothing is ideal so that nothing is really on mass shell?

    Also, your point about Wikipedia articles is true, but I’m still interested in whether the opinion expressed by the writer is to be taken at all seriously. I don’t know how well the theory of quantum chromodynamics agrees with the data or whether QED doesn’t work so well in explaining the behavior of bound states. I was hoping a theoretical physicist working in this area could give me a response based on his far better acquaintance with the empirical tests, etc.

  157. Professor Strassler
    I have been reading up on virtual particles lately and have found your article and one by John Baez particularly (no pun intended) illuminating. I have also been reading related articles in Wikipedia. One of these is entitled “Static forces and virtual-particle exchange”. In it I found the the following statement:

    “The virtual-particle formulation is derived from a method known as perturbation theory which is an approximation assuming interactions are not too strong, and was intended for scattering problems, not bound states such as atoms. For the strong force binding quarks into nucleons at low energies, perturbation theory has never been shown to yield results in accord with experiments[2], thus, the validity of the “force-mediating particle” picture is questionable. Similarly, for bound states the method fails[3]. In these cases the physical interpretation must be re-examined.”

    Is this at all accurate, do you think? I was very surprised to read it, because it seems it is questioning the entire Standard Model with the exception of its applicability to scattering experiments. Questionably applicable to the strong force, the weak force, or bound states generally? Wow.

    Also, this question of what is real and what is virtual (the same thing as “not real”?) has been puzzling me, especially because both your article and Baez’s both end up saying something like even real particles are sort of virtual. I had the idea, perhaps influenced by reading parts of John Bell’s book “The Speakable and Unspeakable in Quantum Mechanics”, that photons that enter my eyes and help create the internal image in my head are definitely real and, beyond this, the particle interactions that leave potentially measurable traces are also quite real, but do not require an actual attempt at measurement. Thus, a meteor impact on the far side of the moon leaves a crater even though my current means of transportation will not get me there to inspect it. The virtual interactions and quantum theory more generally are then just some mathematics you apply to estimate the probabilities of these kinds of real events occurring. Wave collapse is associated with such real interactions, but is itself not real, because the wave function itself is just the mathematics too. Therefore, I think there is clear distinction between real and virtual particles, but you and Baez seem to disagree. Where am I wrong?

    1. The article on virtual particles you reference to Baez was not written by him. He merely hosts the article on his website and it is part of a Physics FAQ for the UseNet sci.physics.* groups.

      http://math.ucr.edu/home/baez/physics/Quantum/virtual_particles.html

      Unfortunately the term “virtual” is a poor term for what virtual means in this context. It does not mean “not real”. Even Matt’s description as “disturbances” is a real physical phenomenum. In particle physics, “virtual” simply means “off mass shell” when used in conjunction with specific particles.

      PS. Be careful about Wikipedia articles; most anyone can edit them at any time.

  158. Dear Professor Strassler,

    I have thought of an analogy regarding the virtual particles and Higgs field. But I would like to know if it is valid or lacking. As I understand it, the Higgs field is a condensate of virtual particles that have become real. These virtual particles, known as the “Dirac sea”, have a negative energy (relative to the vacuum of space) and became real because it was energetically favorable to do so (similar to how electrons act in super conductors?) due to spontaneous symmetry breaking of the vacuum (I do not entirely understand this concept.)
    Every day we go outside, and know there is water vapor in the air. But without our senses to feel this on a humid day, we would be none the wiser to this water vapor. But as the sun sets (spontaneous symmetry breaking of the vacuum) and the next morning we wake to find dew on the grass. The water vapor represents the virtual particle sea and the dew, the Higgs field. At which point, the weak force bosons and other particles affected by the Higgs field are observed to have their mass due to interaction with the “dew”.
    Finally, thank you. This site is incredibly helpful and I have recommended it to several friends, all of whom have enjoyed your writing. We all give our thanks to your work.

    Sincerely, S.B.O.

  159. dear sir
    thank you for your answer,but i have equestion if you please
    in compton effect we have an elasticcollission, and we know very well the laws which govern it, but in photoelectric case the electron absorb the photon, what kind of interaction is the absorbtion, and in the laser case the photon stimulate the electron to emit a photon, what kind of interaction is the stimulation, in the three cases the interaction is between electron and photon, which must be the same in the three cases, but every case has different interpretation, do you think this is acceptable.
    regards
    hosam otaibi.

  160. dear sir
    thank you for this enteresting article, and i have some equestions
    1-after years of discovering the photon, do you think that it is a well known concept in physics.
    2-if we have experiment and we succeed to explane it by mathematical equations, is it nesessary that our explanation is correct.
    3-when a photon interacts with electron, is the direction of every one before and after the interaction is taken in acount in our calculations, for example: compton effect and photoelectric effect.
    thank you.

    1. 1- 100% well-known. Photons are used daily; they’re part of modern technology. The CCDs in digital cameras, for instance, absorb them. Lasers only work because one can make photons behave in lock-step, giving a beam of light that’s narrow and has a single color.

      2- generally multiple types of explanations give the same equations; even if the equations are right that does not mean that the conceptual explanation is unique. Newton’s laws are different in word form from the Hamilton-Jacobi equations and from the minimum action principle, but they lead to the same equations.

      3- everything is taken into account; the predictions of compton and photoelectric effects are extraordinarily precise, to better than one part in 10,000.

  161. Hi Dr. Strassler,
    I’m very happy I found this website! I don’t have any professional education when it comes to physics, but I enjoy learning about various parts of physics on my own. I think your explanations are great!

    In my “study” of virtual particles, this article helped me understand a lot of things. Assuming I understand correctly, it seems physicists used the energy-time uncertainty relation (derived from the uncertainty principle), to predict that virtual particles of a certain energy could come into existence for a time proportional to that energy, and thereby act as force carriers. The range of the nuclear interaction was already known, and from this range physicists (Yukawa, was it?) were able to calculate what the mass of the associated virtual particle should be. Later on, a particle with close to the predicted mass was discovered (the pion).

    This made sense to me when I read it. Your explanation here of virtual particles (field disturbances) also makes sense, but at this point I can’t see how it would be used to predict or explain specific phenomena (such as pions). Can you write a bit about this?

      1. Well, ideally I’d appreciate if you could answer my question above 🙂
        But also, I have questions I’d like to ask on other posts of yours, as well, and if certain questions are more likely to get answered than others, I’d sincerely like to know what those criteria are!

  162. Can the figure 3 be developed even further? Could the virtual photon there be considered also as a disturbance in a quark-field? If so, could it mean that every particle/wave contains virtually all other particles/waves, in a row of infinite implication?

    1. The virtual photon is a disturbance in the electromagnetic field (i.e. the photon field). That’s always true, by definition: a virtual particle of any type is a disturbance in its corresponding field, not in some other field.

      But it is true that all fields affect each other to a greater or lesser degree, so a particle — naively a ripple in its corresponding field — is also made partly from disturbances, to a greater or lesser degree, in other fields.

  163. Professor Strassler,
    In your opinion are “continual spontaneous quantum field disturbances,” “quantum fluctuations,” “zero-point energy,” and “virtual particle/antiparticle annihilation,” different words for the same phenomena or are they different phenomena? If they are different what are the differences?

  164. That was very illuminating professor. Thank you for your quick response. I have a few follow up questions.

    1. Your very last statement. “for most disturbances that happen to sit on top of the horizon, their outside-parts are not made from real particles and will not escape the black hole.”…do you mean that most disturbances that happen to sit on the top of the horizon have a virtual particle form and an anti-virtual particle form that quickly annihilate each other without entering the event horizon. Or, that both the virtual particle form and ant-virtual particle form will both fall into the event horizon? I cling to the former.

    2. Regarding “the very rare thing”. At the event horizon, “the continual spontaneous disturbance”, is usually “white noise” (my term), or a virtual particle and anti-virtual particle that quickly annihilate each other without falling into the event horizon. Can it be assumed, that in very rare events, the virtual particle and anti-virtual particle or “white noise” somehow achieve a “harmonic” (my term) and in essence become a real particle and real anti-particle, where the newly formed, anti-particle falls into the event horizon, shrinking the mass of the black hole, while the newly formed, real particle escapes the event horizon?

    I have a flurry of other questions on the subject…but for another time.

    1. 1. This is an observer-dependent statement, and that’s why I was nervous about giving you the answer I did. I am thinking I should revise it, because there are actually two cases, and I didn’t state them both…

      In any case, the latter is more accurate in one of the cases; from the outside observer’s point of view, the disturbance falls into the horizon; from the point of view of an observer falling into the horizon, it’s just an ordinary disturbance like any other. The former happens too, but those aren’t the interesting disturbances that have something to do with the black hole’s hot atmosphere.

      2. The process is Black Hole –> Smaller Black Hole + real particle. Whether the anti-particle was real cannot be ascertained by any experiment; it’s just a more general disturbance falling into the black hole.

      I’m really not prepared to do this subject justice without pictures and careful thought about how to explain this clearly. It needs articles, not ad hoc answers, I’m afraid. And I certainly have to keep focused on other things for now. But I understand the interest in clear exposition of the subject. Most of what I know I learned from Lenny Susskind himself, so you might look at videos he’s made to explain black holes to the public. But I don’t know how thorough he was.

  165. Professor Strassler, could you explain Hawking radiation from your perspective? Or, do you feel that it is not a correct theory?

    1. You mention that spontaneous disturbances (quantum fluctuations) usually reconnect. I always wondered how they meet again? (Regardless of discussions about black holes.)

      If these disturbances, for instance, start off in opposite directions, would they not need to turn around exactly 180 degrees at some point, and go back to reach each other again? That sounds like a fairly improbable event. (Even without a black hole close by.) What is wrong with this picture?

      1. This is another reason why it is a bad idea to think of these disturbances as “particles”; if you created a pair of particles traveling in opposite directions, why would they ever turn around and annihilate again? This is just the wrong intuition; it just doesn’t work that way. The disturbances can do all sorts of things particles can’t do, such as appear without the input of any energy at all. I take the point of view that one is better off thinking of a quantum field as a seething, complex object, rather than thinking about particle pairs appearing and disappearing in processes that do indeed sound like they violate conservation laws.

    2. I’ve tried to imagine the quantum field as a “seething, complex object” too, like an ocean surface with just random waves going everywhere, that just average out all together, rather than having to pair up the same way they appeared.

      But with that picture, on the other hand, it is not clear how it would make any difference for one “part” of a disturbance if another random “part” would become trapped inside a black hole so they can’t “reconnect”. It also
      becomes less clear how some “part” of a disturbance then can start traveling far away. Maybe something is still missing in this intuition? (Too much of the opposite intuition?)

      (And what are these “parts” you talk about in a disturbance, really? You still seem to talk about them yourself like objects of their own, and not just as one big complex field…)

  166. There are many senses in which it does not make sense to treat the electromagnetic field around the electron as classical, but one of the worst arises in the context of the gluon. The gluon is a particle, like the electron. But gluons interact with gluons; the gluon has a chromoelectric field, like the electron’s electric field. And the chromoelectric field is another word for the gluon field, just as the electric field is another word for the photon field. There is no theory where you somehow treat the gluon particle as quantum mechanical and treat its field as classical — this is a completely inconsistent thing to do.

  167. Hi Fred D.
    Thanks for looking more deeply into this. I tried reading the forum you suggested but the format is too hard to follow. I thought Professor Neumaier’s position on virtual particles was odd and I’m glad you flushed it out more explicitly.
    Joe S.

    1. See my answer above. It seems Neumaier has some correct things to say about virtual particles, but then leaps to a conclusion that isn’t at all justified (and is surely contradicted by the successful numerical simulations of quantum field theory, where one does not use virtual particles in the calculational technique but certainly treats the fields around particles as fully quantum mechanical, not classical as Neumaier suggests.)

      1. I sure would like to see a calculational technique that could get the correct lifetime for muon decay without a virtual W boson involved in the procedure. You are screwed right out of the gate since you have to have an outgoing muon neutrino that carries off the quantum number for muon-ness and another charged particle with spin 1 or 0 that carries off the charge and can decay to an electron and an electron anti-neutrino. There is only one particle that we know of that fits the bill. A W boson with spin 1 since there are no charged elementary particles with know of with spin 0. And has to be way off mass shell. But it still has all the other quantum properties necessary for the decay to happen.

        1. At leading order in perturbation theory, the case of muon decay is especially and misleadingly simple, since there is no interference in the amplitude. The amplitude contains one and only one pole, so all calculational techniques look the same; the amplitude-squared is given by the square of a single propagator, and no matter how someone obtains the answer you can call it a virtual particle if you want.

          So let’s instead consider something a little more complicated: e+ e- –> photon photon . There are two Feynman diagrams, each with a virtual electron; of course the virtual electron is different in the two cases. Now the final answer for the amplitude-squared does NOT take the form of a single propagator-squared. So if I can write down the amplitude using other techniques, I never have to see the two propagators, and there is no sense in which the result is obtained by summing up two diagrams with virtual electrons.

          Well, in fact those techniques exist. They go back a long way, decades in fact.

          For more complicated processes, for instance e+ e- –> 4 photons, the number of graphs explodes, but the amplitudes do not become wildly more complicated. Rather than cope with all the cancellations that have to happen in Feynman diagrams, you may be better off doing the calculation another way.

          For instance, at tree level you can use something like

          Recursive Calculations for Processes with n Gluons.
          Frits A. Berends, W.T. Giele (Leiden U.). Dec 1987. 41 pp.
          Published in Nucl.Phys. B306 (1988) 759

          In fact this technique is always incorporated in the faster computer calculations of scattering amplitudes, see for instance http://comix.freacafe.de/ .

          At loop level you could use Bern-Kosower from 1991 http://prl.aps.org/abstract/PRL/v66/i13/p1669_1 , though there are more powerful techniques now that build the loop amplitudes from tree amplitudes using unitarity, such as

          One loop n point gauge theory amplitudes, unitarity and collinear limits.
          Zvi Bern (UCLA), Lance J. Dixon (SLAC), David C. Dunbar (UCLA), David A. Kosower (Saclay). Mar 7, 1994. 53 pp.
          Published in Nucl.Phys. B425 (1994) 217-260
          e-Print: hep-ph/9403226

          And more recent breakthroughs have helped a lot:

          Generalized unitarity and one-loop amplitudes in N=4 super-Yang-Mills.
          Ruth Britto, Freddy Cachazo, Bo Feng (Princeton, Inst. Advanced Study). Dec 2004. 35 pp.
          Published in Nucl.Phys. B725 (2005) 275-305

          One-Loop Calculations with BlackHat.
          C.F. Berger, Z. Bern, Lance J. Dixon, F. Febres Cordero, D. Forde, H. Ita, D.A. Kosower, D. Maitre (MIT, LNS & UCLA & SLAC & Saclay). Jul 2008. 7 pp.
          Published in Nucl.Phys.Proc.Suppl. 183 (2008) 313-319
          e-Print: arXiv:0807.3705

          None of these techniques require discussion of virtual particles in any strict Feynman Diagram sense; they really use complex analysis, fact about quantum field theory, and combinations of tree-level amplitudes.

          What’s the point? Virtual particles as they appear as lines in Feynman diagrams are unnecessary. What we want is answers for scattering amplitude calculations. If Feynman diagrams, with their virtual particle lines, will work, great. Often they’re too slow and cumbersome, and other methods exist that make no reference to those virtual particles.

          That does not mean, of course, that there aren’t disturbances in fields (as I’ve defined “virtual particles” to be, in order to make them meaningful) that are responsible for the scattering occurring at all. It just means that trying to give meaning to exactly what the virtual particles were and exactly what they were doing involves confusing a crutch for a leg.

          All of this leaves out the great deal of additional subtlety that arises when you remember that nature does not live by perturbation theory; real scattering amplitudes are all-orders results, not tree-level or one-loop-level results. Put that in, along with gauge invariance, and you will soon find yourself unable to define virtual particles as lines in Feynman diagrams… try drawing the more complicated graphs in muon decay, with virtual photons and virtual Zs running between the muon line, electron line, neutrinos and the W, and then put in some lepton loops that split the Z W W or photon W W vertices, and you will already see trouble.

  168. Thanks for your thoughtful response to my September 2, 2012 posting. Your explanation preserves the “reality” of virtual particles with the understanding that the physical effects of their reality can be calculated with or without the use of Feynman Diagrams. In the context of this discussion, the title of you thesis is truly funny. Again thanks for your time.

    Joe S.

    1. Hi Joe,

      Neumaier’s position is from the notion that he believes the Coulomb field of an elementary charged particle is not quantized but remains as a pure classical field. There was a substantial discussion on sci.physics.research with him about this if you are interested.
      https://groups.google.com/forum/#!topic/sci.physics.research/c-hWst9vu68/discussion

      Of course if you take that particular position, you will have to reject the notion that virtual particles are real. In Matt’s language then the Coulomb field of an elementary charged particle is just one big disturbance. But I suspect that Matt would have it as a disturbance of a quantum field whereas Neumaier has it as a disturbance of a classical field.

      As I mentioned previously in this thread, I was taught that the only difference between a real and virtual particle is that the word “virtual” simply means “off mass shell” and all other quantum properties of the particle remain the same. Though quantizing the Coulomb field is a bit of a tough nut to crack, I simply think of it as the quantum vacuum is a polarizable medium and the Coulomb field (and magnetic fields) are a “tilt” of virtual fermionic pairs due to the presence of the elementary charged particle. In that viewpoint, the Coulomb field is quantized and I think agrees with Matt’s “disturbance”. A problem with keeping the Coulomb field as a classical field is instantaneous action at a distance. So I have to reject Neumaier’s position. Besides all that, if you reject that quantum particles can’t be momentarily “off mass shell” you are rejecting the uncertainty principle.

      Fred

      1. Wow. If that’s Neumaier’s conclusion, then indeed, as Fred suggests, I completely disagree with that!!!!! There is no consistent theory of an electron coupled to a classical electromagnetic field in which the electron can also emit discrete photons in scattering processes, as real electrons in our world do. That’s a physics fact that has nothing to do with whether or not you choose to calculate using Feynman diagrams.

        However, Fred, your notion of virtual particles being the same as real particles but “off-mass-shell” is a perturbative notion that does not survive to the fully non-perturbative theory.

        At best, real particles are poles in propagators (possibly off the real axis) and they satisfy various theorems. The virtual particles do not satisfy these theorems. Another way to say this is that the propagators can take a wide variety of functional forms, depending on how the interactions in the field theory work; but a pole in a propagator has universal properties.

        And even this statement isn’t correct. Most propagators that have the relevant poles aren’t even gauge invariant [for example, the electron propagator isn’t.] You really have to look at asymptotic states of the theory and their scattering amplitudes, very carefully defined. In the end the whole relation between real particles (which are things that really exist and are, for instance, gauge invariant) and any notion of virtual particles (which aren’t in general gauge invariant) completely falls apart.

  169. Hi, Professor Strassler! I’m with Kevin (August 14, 2012), I feel so fortunate to have found your website–in particular your posts on virtual particles. I have the same basic question as Kevin’s regarding virtual particles: Do you know, or can you guess why some people (Physicist Arnold Neumaier, http://www.mat.univie.ac.at/~neum/physfaq/topics/virtual) have written that virtual particles (VP) are merely mathematical artifacts of perturbative quantum field theory, and thus VP have no ontological basis? Should I just ignore those people (I’m being sort of glib)”

    Some quotes from Professor Neumaier:

    “Virtual particles are part of the imagery of quantum field theory. They are figurative language for abstract mathematics, used by experts and laymen as imagery for giving abstract recipes for calculating scattering amplitudes an appearance of intuitive meaning. However, any attempt to take this language literally gives a very misleading and unscientific view of the microscopic world …
    Therefore virtual particles ”exist” as lines on paper, as intuition in people’s minds, as superficial but catchy allusions to images that make abstract things concrete, but not as tangible, verifiable entities. On the level of physics, virtual particles are quite similar to what ghosts are on the level of ordinary experience. One cannot ascribe to them most properties that real things have. One can only ascribe to them the properties of internal lines in diagrams and multidimensional integrals in perturbative computations. Once one attempts to ascribe to them more, one gets nonsense.
    Since virtual particles are defined only in terms of the Feynman diagrams, they describe asymptotic properties
    of the scattering, not an actual motion (which would be described by some process at finite times). Thus virtual particles don’t ”move”. They are ”exchanged”, but it makes no scientific sense sense to talk about their motion, their speed, or about the direction they travel, This is meaningless talk, and asking about such properties is like asking for the speed of a ghost.
    Thus it seems impossible to place the superficial virtual particle picture on a sound scientific footing. It is a picture valid only if restricted to the superficial level where no detailed inquiries are made. It is like ordinary people using the word ghost to describe a fleeting but fear-provoking experience. It makes sense only as long as you don’t ask about their precise properties. But once you start asking how fast a ghost is traveling, things no longer make sense, since the concept of a ghost is not intended to be applied literally.”

    In response to A.J | July 13, 2012 at 9:18 PM | You reply, “What the author (Professor Neumaier) leaves out is that those general effects ARE measured and studied, and in fact are hugely important — Casimir effect, changing of the strengths of forces away from the 1/r^2 approximation, scattering of photons off each other, etc. I’m sure he would agree, he just was making a different point…”

    However, Professor Neumaier writes, “How can anything be real if its existence depends on a particular way of viewing the world? How can an experiment (verifying the Casimir effect, say) can be said to prove the existence of virtual particles if the same experiment can be explained by a method of calculation not involving virtual particles at all?”

    You make virtual particles sound influential and at least potentially real, “a disturbance in a field can become a “real” particle with the addition of more energy.” This had always been my assumption about virtual particles but Professor Neumaeir’s posting thru me into confusion.

    Please help me understand any differences, if any, between your view of Virtual Particles and Professor Neumaeir’s.

    Thanks!
    Joe S.

    1. Hmm… The main difference between me and Neumaier seems to be one of definitions, not substance. At least I think so. [I was quite wrong — see Fred’s comment below. Neumaier and I agree virtual particles as lines in Feynman diagrams are fictions, but the conclusion that he draws, which is inconsistent with quantum field theory, I completely disagree with. And so does data from lattice simulations of quantum field theory, which makes no reference to virtual particles, gets the right answer for ratios of hadron masses, but certainly disagrees with Neumaier’s conclusion.]

      Disturbances in fields are real. I *think* Neumaier would agree. The Casimir effect proves the existence of quantum disturbances of fields that are not particles. So do atoms and other bound states, through both their existence and the subtle effects that one must account for to get their binding energies correct.

      Now the question is: do you call those “disturbances in fields” by the name “virtual particles” or not? I think Neumaier is taking a strict perturbative Quantum Field Theory definition of what a virtual particle is (a line in a Feynman diagram) and so he would say (I think) that disturbances in fields may be real, but expressing those disturbances in terms of lines in Feynman diagrams (which is a *calculational technique*, and not necessary) involves introducing non-real artifacts. I agree with that statement. He happens to call the “lines in Feynman diagrams” by the name “virtual particles”. Strictly speaking that’s what one should do in a technically accurate and precise context. So I don’t disagree with his approach.

      I took a different line of approach. I said: Feynman diagrams (and the lines in them) are a calculational approach, but disturbances in fields are real. Since the name most non-experts know for “disturbances in fields” is “virtual particles”, I decided to stick with the name “virtual particles” but redefine it slightly so that it actually does mean something physically real.

      The merit of Neumaier’s approach is philosophical precision with very clear definitions. The merit of my approach is that the words “virtual particles” are widely known among non-experts (and widely used in the field and when talking to the public) and they aren’t going away. But the two approaches agree that the calculational tool known as Feynman diagrams — and the specific way that virtual particles appear there — are not representative of something real, because the calculation can be done in other ways.

      I think the two approaches also agree that in a physical process, certain fields are disturbed a lot more than others — and that this statement can be assigned reality because no matter how you do the calculation you’d come to that conclusion. For example, we may or may not choose to calculate the shift of the magnetic moment of the electron using Feynman diagrams and the specific way that virtual particles appear there. But I think we’d agree that a disturbance in the electromagnetic field is responsible, because the result of the calculation (no matter how you carry it out) is proportional to the strength of the electromagnetic force.

      Amusingly, the title of the paper that became my Ph.D. thesis was “Field Theory Without Feynman Diagrams.” http://arxiv.org/abs/hep-ph/9205205

  170. Martin: “Why the different particles have different ranges? In this matter I’m not 100% sure, but in case of the gluons (which are massless) it is probably due to the confinement because of the color charge.”

    I do know what confinement means. But that is just a word. It does not give an answer to my question. If the fields are around everywhere and all the time, what determines (and why) the different ranges? Weak force is weak and has a short range because of massive bosons (that´s easy), but why is strong force strong and has a short range although gluons are not weak and massless. So how does the “confinement” do the trick?

  171. Hi Matt, I’m a theoretical particle physicist PhD that ended up in biophysics for 20 years. What’s your thoughts on these virtual particle conundrums:
    1. If virtual particles aren’t real (in some sense) why do calculated probabilities depend on properties of virtual particles (eg B0-B0bar mixing box diagrams depend on top quark mass etc)? Although one can say they are ‘integrated out’ that’s not entirely true, the processes depend on virtual particle properties like mt. And, forgive me, I have not been able to read all of the above entries. Related, how would a non-perturbative treatment of B0-B0bar mixing hope to involve the top quark mass? If virtual particles are in some sense real, what sense and what do they mean non-perturbatively?
    2. A fun one here re-quantum gravity. I’ve never had enough time to get into this. Would quantum gravity do away with curved space time or simply account for it?
    3. How would virtual gravitons (in a calculation) escape from a black hole so that the black hole could, act like a black hole, and suck things in? Presumably their virtualness . .
    Regards – Paul

  172. Hi, Mr. Strassler! I feel so fortunate to have found your website–in particular your posts on the composition of protons and explanation of virtual particles. I have a question regarding virtual particles: Do you know, or can you guess why some people have written that virtual particles (VP) are merely mathematical artifacts of perturbative quantum field theory, and thus VP have no ontological basis? Should I just ignore those people (I’m being sort of glib)? By the way, I’ve also read articles by Frank Wilczek which generally correspond to what you have written about VP. Can you set me straight? Thanks!

    Kevin

  173. Strassler: “Yes, all fields related to the known ELEMENTARY PARTICLES are believed to exist everywhere in space, and at all times.”

    Martin: “You are probably confusing fields with particles. The fields exist all the time everywhere. Particles are something that may or may not appear in a field and may have limited range or life time. But the background field is always there. No matter if any particles run through them. I hope I got it right”.

    So,sorry I am milking (don´t get nervous): does for ex the Z-field exist ” everywhere in space, and at all times”, although there are no Z-particles in the vicinity? Or the gluon field? If the answer is, that in spite of the range all fields always exist everywhere in space, so why do the different particles (“forces”,fields?) have different ranges? The answer can not be mass of the interaction particles, because gluons are massless. I can understand why weak force is weak (and has short range); the Z and W particles have big mass, they are lazy.

    1. I’m no expert, so I’m just telling you what I understood from Professor’s explanation: yes, the Z-field exists everywhere in space at all times, although there are no Z particles anywhere. The only important thing: the value of the field is zero (on average). The field does exist, but is zero, until you disturb the field somehow (through a disturbance in another field that interacts with the Z-field).

      Why the different particles have different ranges? In this matter I’m not 100% sure, but in case of the gluons (which are massless) it is probably due to the confinement because of the color charge. Weak force has limited range probably because its bosons are very massive and they decay quickly into other particles. Photon does not have any such constraints, so it can get to great distances.

  174. Agreed. The laws of nature are Lorentz invariant, and there is no preferred inertial frame. But previous posts state that “[These] fields exist all the time everywhere.”

    So what kind of fields can be Lorentz-invariant? Can a Higgs field, for example, be Lorentz invariant?

  175. So fields exist everywhere all the time. These fields have to be Lorentz-invariant, so any observer in any inertial (non-accelerating) Lorentz frame would always see the same field. This would not be true of either individual electric or magnetic fields because an E field Lorentz-transforms to a B field, and vice-versa. So how can these fields be Lorentz-invariant?

    1. Because in vacuum, the electric and magnetic fields are zero. And zero is Lorentz invariant.

      Only the vacuum, and the laws of nature in vacuum (in regions that are relatively small compared to the universe as a whole) are Lorentz invariant.

      Indeed, if there is a magnetic field in your vicinity, Lorentz invariance is locally broken. However, then you have to look at physics more carefully, because the effect of Lorentz violation may or may not be important on experiments that you are doing. For example, there are magnetic fields inside the Large Hadron Collider [LHC] experiments, and those break Lorentz invariance. However, they are so (relatively) small that they have only a very tiny effect on the particle-particle collisions that occur inside the LHC, so Lorentz invariance is still true to very high accuracy in those collisions.

  176. You write: “Yes, all fields related to the known elementary particles are believed to exist everywhere in space, and at all times. There is just one electron field that is everywhere in the universe. What makes the Higgs field different from the other known fields is that it is non-zero on average everywhere in the universe, while, say, the photon field [i.e. the electric and magnetic fields treated together] and the electron field are on average essentially zero”.

    Question: take for ex. the Z-field (or other weak force bosons or quarks and gluons). They do not exist “everywhere in space and at all times”. Their range is very limited. Or did I get it wrong (again)?

    1. You are probably confusing fields with particles. The fields exist all the time everywhere. Particles are something that may or may not appear in a field and may have limited range or life time. But the background field is always there. No matter if any particles run through them. I hope I got it right.

  177. You write: ”But there is so much energy trapped inside a proton that there is enough to make those virtual quarks and anti-quarks almost real”. You also write that energy does not make the ripples or disturbances (of fields), but that the fields (which are fundamental) contain energy: “A field is one thing; energy is something it can have, but there is no such thing as an energy field”. “Can have”; so are there fields containing no field energy? Or are these two things always married? “A particle has energy through its mass and through its motion”. How about a field containing no particles and not moving? I do understand there is no such thing as “pure energy”.

    Or did I get it right? Does energy make the ripples (and disturbances)? If not, who does? Who creates the sine (or not-sine in disturbance)?

  178. so, if I got it right, by ripple you mean a specific kind of oscillation in the field, with values that come out of a set of equations, a stable one; while by disturbance you mean another kind of oscillation, that has different values and is not stable. Are there other peculiar properties that are different between particles and virtual particles besides stability?

    As for the Decay process, I was thinking of something like W boson in muon decay http://en.wikipedia.org/wiki/Particle_decay , what I’d like to know is how the resonant ripple turns to other things in this view. By “how” I mean how you describe the process, what does it mean that a particle can turn in other particles in terms of a wave turning into other waves or combinations of waves.

    1. You’ve only partly got it right; but to really get it right we do need to do math. It would be very easy to explain with equations.

      “Virtual particles” is really a term that has multiple meanings capturing many non-particle processes in quantum field theory, and even if I used math I’d have to give you quite a few examples. Particles, on the other hand, are very easy to describe. I simply would not ask your question “are there other peculiar properties that are different between particles and virtual particles”; there’s almost nothing about “virtual particles”, viewed in general, that is similar to the corresponding particle, other than that they carry the same electric charge and other similar conserved quantities [but that’s just because the charge is something the field has, and the particle, as a ripple in the field, inherits it.]

      Do you understand my example of the swing, and the difference between a resonant and non-resonant process?

      The W boson in muon decay is not a particle; it is “off-shell”, which is technical jargon for the statement that it is representing a non-particle disturbance in the W field. So there is no resonance in the W field in this case.

      How one wave (resonant or not) can turn into other waves is something I plan to explain in the coming weeks, in my series on how the Higgs field works. Please stay tuned for that.

      1. “Do you understand my example of the swing, and the difference between a resonant and non-resonant process?”
        I think so! the resonant process should be (like?) a stationary wave, right? While the non-resonant is a wave that is not stable in the medium it travels in and will rapidly degenerate, eventually leaving a static wave behind.

        “How one wave (resonant or not) can turn into other waves is something I plan to explain in the coming weeks, in my series on how the Higgs field works. Please stay tuned for that.”
        I’ll try, if I remember of anything when back from holydays 😉

  179. Dear Mr Strassler,
    1st THANK YOU! I love this explanation, I got the whole fritjof capra’s “tao of physics” wrong and you explained me a fundamental bit of information that helped me realizing this.
    2nd I still need a bit of information to understand what you explained: what you mean by “ripple” and “disturbance” in a field? What makes a “ripple” in a field a particle that does not the same of a “disturbance”?
    3rd A topic-related question: How decay processes can be described in terms of field disturbances?
    4th I just read “particles are resonances” in the comment above, what does it means?

    Please notice, if you are going to answer (in which case I thank you again), keep in mind that I’m not familiar with the math behind quantum physics, I studied only basic physics in a computer science bachelor. In other words I do not speak FourierTransformish nor DoubleIntegralese and know only some words of Derivish 😉

    1. Derivish! My favorite.

      Roughly: a ripple is a sine wave, a solution to some form of wave equation. A disturbance is not a solution to the wave equation; it is a solution to an equation which has the same form on the left-hand side of the equation but has something non-zero on the right-hand side.

      A resonance is something that is easy to make: you know that no matter how you hit a tuning fork or bell, it will always ring with the same tone, i.e. vibrate with its resonant frequency. A disturbance, in my language, is a non-resonant process; for instance, if you try to make a tuning fork or bell vibrate at a frequency which it doesn’t prefer, or make it shake in some non-repetitive way, you can do it, but you have to work hard. If you do anything particularly energetic in the universe — say, set off a supernova — you will make lots of particles; you will be much less likely to make all of the more general types of disturbances, and to the extent you do, they will turn into particles very quickly.

      Decay processes are not necessarily related to field disturbances — can you clarify your question? What example are you thinking of?

  180. Hi Matt,

    I am glad you think that virtual “particles” are real. There seems to be a movement in some European circles that they are not real. For example, I got booted off physicsforums.com for arguing that they are real and you can see the link above to Arnold Neumaier’s FAQ (a PF.com advisor). I sort of get your drift that you want to call them disturbances but my particle physics instructor, Dr. Andy Inopin, taught me that they have all the same exact properties as their “real” counterparts except they are simply “off mass shell” and can’t be detected. Well.. of course if they are detected, they become “real”.

    Take muon decay as an example. If there isn’t a real virtual W boson involved, the muon could never decay. That virtual W boson has all the same properties as a “real” W boson except for it is “off mass shell”. So your “disturbance” here sure smells like a W boson particle. 🙂 Now, I think that with lower energy Coulomb type interactions, the “disturbance” connotation could be appropriate. Perhaps you mentioned that above as I did not read all of the preceding comments.

    Best,

    Fred

    1. You are putting huge numbers of words in my mouth here. Your example of the W boson in the case of muon decay is exactly opposite to what I said. You say “your disturbance sure smells like a W boson particle”. Obviously you have a serious problem with your nose. I said very specifically: “`virtual particles’ are NOT particles”; particles are resonances, virtual particles are not. They can even have negative mass-squared (does this not bother you?)

      1. I know it is opposite of what you said. I suspect most particle physicists would disagree with you about the example of muon decay with the virtual W boson being just a “disturbance”. I would think it is in fact a temporary resonance that has all the properties of of a real W boson except for its mass. So there is a possible counter-example to your “disturbance” hypothesis. There are of course many others. In particle physics, the term “virtual particle” has a very specific meaning. It simply means a particle that is “off mass shell”. Griffiths’ says in “Intro. to Elementary Particles”, “Actually, the _physical_ distinction between real and virtual particles is not quite as sharp as I have implied. If a photon is emitted on Alpha Centauri, and absorbed in your eye, it is technically a virtual photon. However, in general, the farther a virtual particle is from its mass shell the shorter it lives, so a photon from a distant star would have to be extremely close to its ‘correct’ mass — it would have to be _almost_ ‘real’.”

        What is an example of a virtual particle that has “negative mass squared”? I would suspect it may be related to momentum. Thanks.

        1. Your suspicion is wrong. And the fact that you have never heard of a virtual particle with negative mass-squared puts a knife into your credibility. [Check: what is the mass-squared of a photon exchanged between an electron and a nucleus?]

          Go learn some quantum field theory, and don’t bother me with irrelevant statements [such as Griffiths’ correct, but obvious, point] until you understand the main difference between a propagator (which can have any mass and a relatively arbitrary functional form) and a particle (which is a pole in that propagator, has a definite mass [possibly with a non-zero width] and is controlled by theorems that do not apply to the propagator.) The fact that two things are continuously connected does not make them equal; yes, the distinction between a virtual particle and a real particle is only sharp if the particle has an infinite lifetime, and even then, there is a limit you can take where one resembles the other (as in Griffiths’ example) and that’s even critical in doing calculations; but no, that doesn’t mean you should view them as simply equivalent. And I’m trying to explain physics to the public on this site, not to experts, for whom this subtlety would need to be properly explored.

          On the example of muon decay; let’s take, instead, the scattering of an electron off a positron to make a W+ and W-. There are three interfering processes, one with a virtual photon of positive mass-squared, one with a virtual Z of positive mass-squared, one with a virtual neutrino of negative mass-squared. Are they all particles? Or are some more “particle” than others? Oh, and that’s only at lowest-order in the quantum field theory. There are also processes where the electron and positron turn into multiple virtual particles with all sorts of different mass-squareds that we have to integrate over. Are these all to be thought of as particles, even though within the integration the mass-squared will vary from negative across zero to positive? And there are several interfering processes, while we’re at it; how are you thinking about these?

          It’s a mistake to view these things as particles. A fundamental mistake. The W particle that appears in top quark decay is different from the W virtual particle that appears in muon decay, or that allows a neutrino to scatter off a nucleus and turn into an electron. The issue is the lifetime. A W particle has a lifetime that corresponds to a width of about 2 GeV. A W particle that is much further off-shell than 2 GeV decays away not because of damping but because it is non-resonant; a W particle that is much less off-shell than 2 GeV decays away due to damping, not due to being off-resonance. Again, these are continuously connected, but so is a young man and an old man; so are red and yellow; that does not make them equivalent.

          I trained under Peskin, Susskind, Banks, Seiberg, Witten, Shenker, and Wilczek, and my research in quantum field theory and my teaching of quantum field theory’s many subtleties is well-known. My readers don’t need your advice.

      2. Apparently Fred D. would prefer to jump to the conclusion that he was censored rather than investigate the comments section of this site a bit to test his hypothesis. Had he put in the time and effort to follow the scientific method and inform himself of the evidence at hand, instead of skipping the data-gathering process altogether and mistaking his layperson’s ignorance for expertise in the field of site moderation, he’d have discovered that all nested replies here (not just his) go only two layers deep. If the work required to become an expert in this area is too daunting for him, he’d do well to listen to those who’ve devoted their careers to doing just that.

      3. @Matt,

        Yes, sorry about that as I realized WordPress has a defect in the way their thread reply works after I posted my reply on SPF. I am used to the FQXi blogs that have the reply link properly at the end of the thread you want to reply to. I have posted a correction to SPF.

  181. Hi,
    You say “forget about the virtual particles, it`s just that the particle disturbs the field”. My simple question: HOW?

    1. The way nature works is that fields interact with each other; a particle is a ripple in one of the fields, and the interaction of that field with other fields disturbs them. There’s no “how” to that — it’s simply what nature does. You’re asking me to define the fundamental processes of nature in terms of even more fundamental things — well, as far as I know, there’s nothing more fundamental than this.

      I can write some equations that show you how this works. But I can’t tell you something more basic than the interactions of fields with one another.

  182. Dear Professor Strassler,

    I was reading on virtual particles, and came across an interesting thought. There is this “sea” of virtual particles at any given point in the vacuum. While I understand the concept that really the only difference between real and virtual is a very subtle one, on whether we can “see” it or not, and if it does or doesn’t violate the conservation laws. From this idea I have four questions- If an electron spits out any virtual particle of sorts, wouldn’t that electron itself become virtual momentarily because two objects bound together cannot split apart without energy from outside the system, even if this effect is immeasurable? Secondly, Dirac predicted the positron with these concepts, virtual particles as pairs. Is this pair of particles actually a pair, or is one merely the appearance of existence of the newly turned “real” particle’s anti-particle, because, when observing a system of, say electrons, if one electron leaves it appears a positive charge is now present? Thirdly, For these virtual particles to become “real” a transfer of energy must occur from a real particle to the virtual particle, so is it possible that a real particle that hit a virtual particle and gave sufficient energy to make them now both real, could lose energy as it travels through the vacuum of space? Is this possibly what we observe as mass, with particles interacting with the higgs field? I also read about how real particles can condense in a vacuum from virtual particles if arranged just right. What does that mean, how do they have to be arranged? If this is true, is it because of Einstein’s binding energy concept? Thank you very much for your time. It is very much appreciated.

    Sincerely, Stone Oliver

  183. Professor Strassler. Thank you very much for your reply.
    So could I then say that a particle is a real particle only if its invariant mass (E^2 – (pc)^2) is its real mass^2, and a virtual particle if its invariant mass is not its real mass.
    Earlier, Neil Fazel | July 15, 2012 at 12:03 PM | replied:
    “I think you need to distinguish between virtual particles which show up as internal states of Feynman diagrams (i.e. in perturbation theory), and virtual particles as “disturbances in a field” (e.g. in vacuum polarization). The virtual particles in the latter category are very much real. The virtual particles in Feynman diagrams are mathematical constructs.”
    With respect to the electron – positron loop (vacuum polarization) in the external Coulomb field (the Uehling correction), are the electron and positron in the loop both real particles as Neil Fazel states, in the sense that their individual invariant masses are both 0.511 MeV? If so, wouldn’t they be free particles, like in pair production? Thank you.

    1. To pre-empt any confusion, I used the term “real” to mean that it is something more than a mathematical construct. If I understood the Professor correctly, the electron and positron in vacuum polarization represent (real) “disturbances in a field”, but not particles. But if, somehow, the disturbances acquire enough amplitude in a short enough time, they could become real particles.

      1. Does it mean that the virtual particles in Feynman diagrams do not represent any disturbances in a field? From what I understand, they represent mathematical terms in “perturbation expansion”. But it should be an expansion of something. For example the terms in Fourier expansion, when added together, they form a function. So there should still be some “disturbance in a field”, even in the case of the Feynman diagrams. However I have never seen these things really written down mathematically, so I don’t know. Is my understanding confused?

  184. Professor Strassler-
    Thank you for this illuminating reply. I do have a question about your statement concerning the relationship between a free electron, and a virtual electron and a virtual photon, as shown in your Figure 4 on the Feynman diagram in your original post. A real electron, regardless on how fast it is moving, has an invariant mass (squared) of E^2 – (pc)^2 = (mc^2)^2 = (0.511 MeV)^2. Similarly, a photon has an invariant mass of zero. Do the real electrons on both sides of the virtual particles in Figure 4 both have an invariant mass of 0.511 MeV? What about the invariant masses of the virtual electron and photon in Figure 4? Are they still 0.511 and zero MeV respectively, or does the combination of the virtual electron and virtual photon preserve the invariant mass of the free electron? Thank you.

    1. Virtual particles do not have the mass of the corresponding real particle. A virtual particle can have any mass, including a negative mass-squared (i.e. a mass that is imaginary) by having more momentum than energy instead of the other way round.) It’s a very bad idea to think about a virtual particle as being something like a particle; it’s not a particle, it’s a generalized disturbance in a field, and it doesn’t obey the rules particles obey. Energy and momentum are conserved in Feynman diagrams at each vertex in the diagram, so the combination of the virtual photon and virtual electron into which the real electron has dissociated has the same energy, momentum and invariant mass of the real electron that entered and exited the diagram.

  185. Hello Professon Strassler,
    Thank you for a great article. I have a question if these messy disturbances (formerly known as “virtual particles”) can be weaker than real particles. In the text about Higgs boson decay into 2 Z bosons I found that one of the Z particles must be virtual (because 2 real Zs are heavier than 1 Higgs). But the “virtual” Z is still somehow there: it is a disturbance in the Z field. So the Z field is disturbed and there is something like a “fraction” of Z boson? Something lighter than a real Z? That would imply that elementary particles are not the dimmest possible waves in the field. But I’m probably misunderstanding something.

    My second question is about the statement that electron spends part of its time as a combination of photon and electron disturbance. Are there some clearly definite separate times when the electron is only electron and then for a nanosecond it is a combination of disturbances? Or the electron is somehow always someting like a quantun superposition of all these variants (i.e. pure electron, combination of electron and photon, combination of other things disturbed by various fields)?

    1. For quanta (i.e. the things we somewhat misleadingly call “particles”) you should not confuse “lower in energy” with “weaker” or “dimmer”. There’s no connection.

      A wave has a frequency (how often does it wiggle) and an amplitude (how far does it wiggle.)

      Particles are lower in energy if they have lower frequency.

      A real particle is the wave of lowest possible amplitude (in a corresponding field). If it is at rest, it has its minimum frequency and energy, corresponding to its mass. If it is moving in some direction, it has a higher frequency and energy. No matter what, its amplitude is the smallest allowed.

      Understanding this is key to understanding Einstein’s insight that led to his explanation of the photo-electric effect.

      A generalized disturbance (corresponding to one or more virtual particles) doesn’t have to satisfy any of these conditions.

      In the end, the electron is the electron. You can write it as a quantum superposition of a free non-interacting electron, a free non-interacting virtual electron along with a virtual photon, a free non-interacting virtual electron along with two virtual photons, a free non-interacting electron along with a virtual muon-antimuon pair, etc. So the electron is always partly all of these things, and no, there aren’t special times when it is one thing rather than the other.

      1. Thank you very much for the response. So the virtual Z in Higgs decay has lower mass (energy, related to its frequency) than real Z (when being at rest). But the virtual Z does not have smaller amplitude than real Z.

  186. Hello there. A theoretical physics website has this to say about virtual particles:
    “If virtual particles were real, they would leave their trace in all methods of predicting certain phenomena, and they would assign the same properties to the virtual particles no matter which approximation method is used.

    However, the literature readily shows that the details of Feynman diagrams strongly depend on the perturbation scheme used: In light front calculations, one gets a completely different set of diagrams than in the more traditional covariant form. And in nonperturbative approaches such as lattice gauge theory or conformal field theory, the predictions do not involve virtual particles at all.
    The nonexistence of virtual particles in nonperturbative calculations (whether conformal field theory or lattice gauge theory) is proof that the virtual particle concept is an artifact of perturbation theory. Something whose existence depends on the method of calculation cannot exist in a strong sense of the
    word.”
    http://www.mat.univie.ac.at/~neum/physfaq/topics/virtual

    What do you think of this?

    1. From A.J. ““If virtual particles were real, they would leave their trace in all methods of predicting certain phenomena, and they would assign the same properties to the virtual particles no matter which approximation method is used.”
      Pair production of electrons and positrons is a well-known component of gamma-ray attenuation (in addition to Compton scattering) above 1.02 MeV, and has been used to produce positron beams from high energy bremsstrahlung since the late 1950’s. Since 1965, the best estimate of the pion mass has been based on measurement of the x-ray transition energies of negative pions bound in atomic levels of various elements (Prior to this, the pion mass was estimated by measuring lengths of tracks in nuclear emulsions). Is the i/r-squared radial dependence of the Coulomb field assumption valid in pionic atoms, when the pion is so close to the nucleus?

      In 1935, Uehling (Phys Rev Vol 48, page 55) showed that the Coulomb field near a nucleus varied from the expected 1/r-squared radial dependence by a new term which he called “vacuum polarization”. Certainly in pionic atoms, there is not enough energy in the atomic binding energies to create electron positron pairs, so what is this “vacuum polarization” term proposed by Uehling? Uehling states;
      “According to Dirac’s theory of the positron, an electromagnetic field will, in general, induce a charge and current distribution due to the creation and annihilation of electron-positron pairs. The induced fields produced by the electron-positron distribution may be regarded phenomenologically as corresponding to supplementary terms in Maxwell’s equations. Since one must demand the validity of [Maxwell’s] equations in sufficiently weak and slowly varying fields, ………. these these [supplementary] terms must depend on higher powers of the field intensities, …….. whenever the fields vary appreciably in a distance of the order of h-bar/mc [electron Compton wavelength], under which circumstances an appreciable polarization electron-positron distribution can exist.” Uehling refers to this as “vacuum polarization”.

      This vacuum polarization proposed by Uehling is a very large correction to the atomic energy levels in pionic atoms, and represents a many standard deviation effect in the pion mass measurement. The pion mass can also be measured by looking at the kinematics of pi-plus 2-body decay into a muon and neutrino, and this confirms the presence of the vacuum polarization term in the Coulomb field at small distances.

      So the vacuum polarization correction to the 1/r-squared law is real, and can be accurately phenomenologically represented by induced creation and annihilation of electron-positron pairs. Is it just an accident that this correction to the Coulomb field is accurately represented by a pair of induced charged particles that have both the charge and mass of electrons and positrons? Is this “vacuum polarization” just an artifact of perturbation theory? Is there another non-perturbative scheme, as proposed by A.J., that yields the same result? i think not. If it looks like a duck, and quacks like a duck, then what should we call it?

      1. I think you need to distinguish between virtual particles which show up as internal states of Feynman diagrams (i.e. in perturbation theory), and virtual particles as “disturbances in a field” (e.g. in vacuum polarization). The virtual particles in the latter category are very much real. The virtual particles in Feynman diagrams are mathematical constructs.

        I think the confusion arises when people draw a particle interaction diagram showing production of outgoing particles from incoming particle via intermediate disturbances in the fields, and call that a Feynman diagram. First of all, the interaction may not even be perturbative, so talking about Feynman diagrams is not valid; and even if it were perturbative, there are an infinite number of Feynman diagrams in the perturbation expansion corresponding to that single interaction diagram.

    2. This is basically right, yes. though it does leave one thing out; see below.

      My statement that virtual particles aren’t particles at all, but are disturbances in fields, is designed to be as consistent as possible with these statements. Those disturbances are very complex. Only in perturbation theory do we have a way of talking about them in a simple way — and indeed, as the author says, there are multiple ways of talking about what they are and how they work.

      Real particles are things you can hold, kick, absorb and study. Virtual particles cannot be isolated and studied — only more general effects of these general disturbances can be studied. What the author leaves out is that those general effects ARE measured and studied, and in fact are hugely important — Casimir effect, changing of the strengths of forces away from the 1/r^2 approximation, scattering of photons off each other, etc. I’m sure he would agree, he just was making a different point… which is that when you express those general effects using nice pictures of particles running around inside of Feynman diagrams, you’re using a particular calculational technique, and not making a statement about the “true nature” of those generalized disturbances. You could have used an entirely different calculational technique and gotten the same answer without ever drawing diagrams of virtual particles running around.

  187. Hi, I recently turned sixteen, so when you spoke about reading of virtual particles and not understanding them at the age of sixteen, my interest was piqued. This article helped solve my curiosity about virtual particles, although I do feel as though I still have a whole lot to learn. Everywhere else I looked didn’t really make any sense, but this did. I just wanted to say thank you. My knowledge is pretty limited, and I was wondering if you could give me links or ideas as to where I could learn more.

    1. Hmmm. Beyond what I’ve told you, I think one has to start getting into the math. If you’re really interested in learning much more, it may be time to sit down and learn some real physics … especially about the phenomena of resonance, and about waves. And probably also about energy and momentum. A freshman physics course is probably what you need (and you can find on-line resources from major universities such as MIT if you’re impatient.)

  188. Following on from Neil Fazel’s point: Do you think that the physics behind quantum electrodynamics is *fundamentally* perturbative i.e. there’s no way even in principle to formulate it “exactly”? My point being that, if you *could* formulate it non perturbatively, then the virtual particle question would never come up. You would have some other way to compute the effects that are currently computed using virtual particle contributions.

  189. I have a quick question: does the “electron field” have anything to do with the probabilistic distribution of the electron…or is this something completely different? I have a certain understanding of what an EM field is, but an electron field?

  190. Prof. Strassler,

    I’m a little confused here. Doing a perturbation expansion is a purely mathematical procedure. But you seem to be saying that the individual terms in the expansion correspond to non-resonant phenomena, with real physical consequences. How do you make this leap?

    1. Well — I see your point. There is a slipperiness here that I’m in danger of letting into my language… there are non-resonant phenomena (start with that) and now you have to figure out how to calculate them. Only at that point do you introduce a mathematical procedure and start drawing diagrams, and in particular expressing things in terms of propagators etc.

      Let me think about what has to be done here, pedagogically.

  191. Here is the definition of virtual particles by http://www.physicsforums.com:

    “Virtual particles are a mathematical device used in perturbation expansions of the S-operator (transition matrix) of an interaction in quantum field theory.

    No virtual particle physically appears in the interaction: all possible virtual particles, and their antiparticles, occur equally and together in the mathematics, and must be removed by integration over the values of their momenta.

    In the coordinate-space representation of a Feynman diagram, the virtual particles are on-mass-shell (realistic), but only 3-momentum is conserved at each vertex, not 4-momentum, so there is no immediate way of obtaining 4-momentum-conserving delta functions.

    In the momentum-space representation, the virtual particles are both on- and off-mass-shell (unrealistic), but 4-momentum is conserved at each vertex, and also round each loop (as shown by a delta function for each).

    In the coordinate-space representation, each virtual particle appears “as itself”, but in the momentum-space representation, it is represented by a “propagator” (a function of its 4-momentum).”

    The statement that virtual particles occur “equally and together” excludes the single off-the-mass-shell electron or positron in the Feynman diagram for Compton scattering.

    1. Your point being?

      This definition is technically correct.

      What it doesn’t do is serve any deeper pedagogical purpose. And it leaves off the physics.

      There are real physical effects from non-resonant phenomena in field theory. Choosing to write those in Fourier notation so that you express them using the “mathematical device used in perturbation expansion” is the technical aspect of how you calculate their effects. But this entirely ignores the physics part.

      The physics part is that these non-resonant phenomena are responsible for:

      electrical and magnetic forces
      scattering of particles off one another
      all sorts of particle production processes
      the shifts in the strengths of forces as a function of distance (beyond the 1/r^2 force law)
      the shifts in the strengths of the electron and muon magnetic moments
      the interaction of the Higgs particle with photons
      and on and on and on…

      So to view these purely as a technical device is not complete; there is an enormous amount of physics here. But to view them as particles is also deeply misleading; particles are a resonant phenomenon. The non-resonant phenomena, for technical and historical reasons, tend all to be called “virtual particles” even though they aren’t particles at all.

  192. I agree that there are two electron-photon vertices in Compton scattering, each with one real photon, and one real electron, and one off-the-mass-shell (virtual) electron or positron. However there is never a virtual electron-positron pair (bubble diagram) in Compton scattering, as suggested by Jack H above.

    1. You are right that there is never a pair of virtual particles; at each step at most one is virtual, at leading order in perturbation theory. But I’m not sure Jack H really meant there was a bubble; I think he meant what I described in my answer. However, I agree his wording was ambiguous.

  193. The total cross section for Compton scattering in the non-relativistic limit is σ = (8 pi/3)(r_e)^2 = 0.66 barns. This is also the total cross section for classical Thomson scattering. If the total cross section for Compton and Thomson scattering included virtual pair production and annihilation, wouldn’t the cross section include a factor.α= e^2/h-bar c ?

    1. No. First of all, r_e is not a real radius of anything; after quantum mechanics we understand that r_e = 4 pi alpha / m_e c^2 . So there is already an alpha^2 in the cross-sections. Second, the scattering process in Compton scattering is described in quantum field theory as

      photon + electron –> virtual electron –> scattered photon + scattered electron

      In an ancient notation (which is what the previous commenter was referring to) you could write this as

      photon –> scattered electron + virtual positron

      followed by

      electron + virtual positron –> scattered photon

      which has the same effect.

      Either way you need a virtual fermion somewhere. (Actually, depending on what frame you use, there’s another quantum contribution; but that too has a virtual fermion.)

      In all of these leading-order processes (to which there are additional quantum corrections) you get a factor of “e” at each “–>”, so that gives you e^2 in the amplitude, and thus e^4 ~ alpha^2 for the probability and the cross-section.

  194. I think you’re selling virtual particles short here.
    One way to think of Compton scattering is that the photon forms a virtual e^-e^+ pair, the virtual positron annihilates the original “real” electron giving off a photon and the “virtual” electron continues out of the Feynman diagram as the scattered “real” electron.
    So we can talk about things being on-shell and off-shell, but to say virtual particles aren’t real is, I think, too simplistic a view.

    1. For some reason you misunderstood me; perhaps you can direct me to the point in the article where my meaning became confusing?

      I did not say (or at least, mean to say) that virtual particles aren’t real.

      What I said is that virtual particles are not particles — they are more generalized disturbances in fields. However, they are very real; they are responsible for all of the basic forces of nature.

      The example you gave (Compton scattering) is completely consistent with this. The “off-shell particle” is most definitely real, but it is not a one-particle state of the Hamiltonian. In fact it only looks like a single particle in first-order perturbation theory. That is to be contrasted with the photon and the electron in the initial and final state, which are one-particle states of the (all-orders) Hamiltonian.

    2. Including a virtual electron-positron pair in Compton scattering makes the total cross section too small, and also does not match well to the Thomson scattering cross section (= 2/3 barn) at very low energies.

  195. Dear professor Strassler

    I am so happy to have found your website. It has been extremely educating. thank you.  I hope you write the book(s). There at least two books: the perspective for the layperson; an undergraduate level for budding physicists. 

    I finished my doctorate in physics back in 1995 and became a patent consultant. My doctorate thesis was on magnetic thin films. I did some modelling work on the experimental results. The model explained a number of things especially interlayer magnetic spin coupling and it made some predictions that could be tested. After Presenting the model at a conference, a friend asked me a profound question: Abu is this what is really happening at the spin level in the thin films? I replied that the model fitted the data well and made predictions that could be tested and where tested. And so based on that it was a valid model. However, regarding his question in the final analysis I have to be honest and say I don’t know.

    Now coming to ‘virtual particles’, ‘quantum fields’, ( quantum mechanics) etc and their underlying models  they are indeed useful, powerful but in the final analysis do they describe reality that actually exists?

    Regarding quantum fields for some reason I always imagined them to be one-dimensional lines: is it better to think of them as volumes – since they fill all space? And is regarding the different (particle, force) fields as being intertwined a useful analogy when thinking about particle decay?
     Many thanks
    Abuisa

    1. In the case of the thin films, we do understand that there is another layer of reality below the layer that you were using to describe the phenomenon — so the question you were asked was well-posed.

      But when you ask “in the final analysis do they describe reality that actually exists?”, the answer has two parts. First, how do we know we are anywhere near a final analysis? There could certainly be more layers of reality beyond the ones we’ve encountered so far. What experiment can you imagine doing that could ever answer your question with finality? And if no experiment exists, is your question a scientific question, in the end?

      Furthermore, what does “actually exist” mean? can you imagine an experiment that tells you whether something really exists or whether it is just a useful tool for describing and predicting the world? Again, if there is no experiment, can this question be answered scientifically?

      As far as we know, the description of the world using quantum fields and so forth is an excellent way to think about the world that allows for a vast array of measurements to be predicted in advance. But we neither know this is a unique nor a final way to think about reality.

      Probably the reason you think of fields in terms of one-dimensional lines is because of what you know about the electric field, which is indeed drawn as field lines. But that does not work for all types of fields. Generally, a field is a function of space and time — start with that. It might just be a number at every point: F(x,y,z,t). In the case of an electric field, it would be a vector at every point: E_x(x,y,z,t), E_y(x,y,z,t), E_z(x,y,z,t); in fact you need to generalize this to include also the magnetic field to make it consistent with relativity, but this isn’t so important here. After Einstein, the gravitational field is a function with even more indices on it. For an electron field, you need a function that is a one-by-four matrix — etc. So this is to say that all fields are functions of space and time, but different types of fields, depending on their spin and perhaps on other properties, will actually be collections of functions, ones that can only be thought of in terms of field lines in the special case that the functions form a vector.

  196. Prof. Strassler,

    Thank you for this article. I have a few questions:

    1) I always thought that, in the context of two “real” particles interacting with each other, Feynman diagrams were a mathematical convenience, with each diagram representing a term in the perturbation expansion, and virtual particles were nothing more than a pictorial way of representing the propagator. You characterize a virtual particle as a “disturbance” in the field; how does one reconcile the “mathematical convenience” view of virtual particles with the “disturbance in the field” view of it?

    2) I have read the “Schwinger limit” described as the electric field of a laser which is strong enough to pull virtual electron-positron pairs out of the vacuum and make them real. Using your language, the laser field creates disturbances in both the electron and positron fields, i.e. polarizes the vacuum, and if these disturbances are large enough, they become ripples in the fields, i.e. a real electron-positron pair. Is that an accurate description of what happens when a strong laser field interacts with the vacuum and creates electron-positron pairs?

    3) How can the following statement from the Wikipedia page for virtual particles be reconciled with your description of virtual photons as “disturbances” in the photon field:

    “Virtual photons are also a major component of antenna near field phenomena and induction fields, which have shorter-range effects, and do not radiate through space with the same range-properties as do electromagnetic wave photons. For example, the energy carried from one winding of a transformer to another, or to and from a patient in an MRI scanner, in quantum terms is carried by virtual photons, not real photons.”

    4) At one point you said, “Exactly the same equations that tell us about photons also tell us about how these disturbances work; in fact, the equations of quantum fields guarantee that if nature can have photons, it can have these disturbances too.” Could you be more specific? I am trying to relate what you said here to what I have read in QFT books. How does your description relate to the S matrix, propagators, and perturbation expansion? I have seen the QFT formulation of electron-electron scattering which leads to a scattering cross section, but how does the repulsive force follow from that?

    Thanks,
    Neil

    1. 1) propagator = Green function = off-resonance disturbance in the field caused by a source. Example: 1/r electric potential between charges.

      singularity in the propagator = resonance = ripple in the field that can travel indefinitely without a source. Example: photon

      2) Yes. If the energy is large enough the singularities in the propagators can be accessed and there is a non-analytic change in the response of the vacuum. This also happens for quarks and antiquarks in a strong gluon field, in the context of jet formation; see http://profmattstrassler.com/articles-and-posts/particle-physics-basics/the-known-apparently-elementary-particles/jets-the-manifestation-of-quarks-and-gluons/

      3) See (1); the near-field effects are off-resonance. Photons are on-resonance.

      4) See (1): the equation for a Green function G for a field phi is O(G) = J, where O is some differential operator and J is a source; the equation for a resonance is O(phi) = 0, for the same operator O.

      Said differently: the “virtual particle” i.e. Green function i.e. propagator satisfies an inhomogeneous linear differential equation, while the real particle satisfies the homogeneous version of the same equation.

  197. I love these articles. Like you, I’ve read many books for laypeople on particle physics (and even glanced into a few for experts), and your articles are the first that have given me even a glimmer of understanding of quantum field theory.

    The explanations of virtual particles I’ve read have generally been along the lines of “the uncertainly principle allows conservation of energy and momentum to be violated over very short times, and virtual particles are the result.” How does this relate to what you say here? Or is it another “white lie”?

    1. Another white lie, but it gives roughly the right estimates (when you apply the uncertainty relations) for how common are quantum disturbances of a given size. That’s why people talk that way.

      1. Could you please explain why it was a white lie? I have been wandering all around the internet and everyone seems to give conflicting views.
        I would be very grateful if you could help.

  198. Hi, do you know how I found this? At a depression website bulletin board labeled as off topic. Some of the smartest people I know are nuts. And of course, I didn’t get there by accident, even though I got here by accident.

    I love this stuff. Total lay person here. I was a surveyor though and I can use sine, cosine and tangent to lay out the corner of the latest Walmart building to within…well, a thousandth of a foot. Close enough for concrete. And using those 3 functions and a 20 dollar calculator and a hundred foot tape, I can check into known points, inverse some rectangular coordinates into angle and distance, and voila! Attention shoppers, Brian has caused all this.

    You write so well…better than Feynman. That’s right, I said it. Q E D (major and minor premises, with a happy ending type of Q E D)
    Thanks, excellent. The analogies, ie. the spring, the swing, are huge, Feynmanesque. My spatial faculties have grown a lobe.

    layman Brian, eating in a corned beef and cabbage field, interacting with a coffee swallow field. It is almost time for a cigarette field. (Did you ever look at cigarette smoke in a sun ray in a overly windowed room? It is 1.) blue. and 2.) made up of tiny particles.)

    1. “layman Brian, eating in a corned beef and cabbage field, interacting with a coffee swallow field. It is almost time for a cigarette field.”

      this made me smile

  199. The importance of virtual particles is in whether they have an effect on measurable quantities.. If they had no effect, then studying them is just a mental exercise. Vacuum polarization of virtual particles is a real effect, and in the case of pionic atoms, the atomic energy levels are shifted by a large amount. The first precise measurement of the pion mass was done by measuring the atomic transition energies of pions in pionic atoms, and calculating the level shift due to vacuum polarization. See Appendix B in Robert E. Shafer, “Pion Mass Measurement…”, Phys. Rev. 163, 1451 (1967) The vacuum polarization effect was first calculated by Uehling in Phys Rev (1935).

    This atomic level shift is related to charge renormalization, in that virtual particles in strong Coulomb field partially shield the bare Coulomb field..
    Robert Shafer

  200. Dear Sir,

    Thank you very much for your great patient and explaination.
    I really do appreciate it.
    So I will just have to change the last part of my statement to “there is a tiny possibility that this space may be created by some form of matter or energy, but we really don’t know what space is made of.”

    Thanking you,

    Dr HW Looi.

  201. Dear Sir,

    Thank you for answering my question. But I think you have completely misunderstood what I was trying to say.

    Of course I know very well that the conventional Einstein concept of gravity is that it bends space-time and as such causes two objects to move towards each other. And the Newtonion concept is that it is nothing more than just an attractive force.

    What I am trying to say is that there maybe another more simple concept.
    If there are 2 objects in the vacuum of space, and if there is really absolutely nothing in between, the two objects do not just move towards each other, but rather they should be next to each other!

    Take another analogy. If there are 2 chairs that are 10 feet apart. There is a “space” of 10 ft and if there is absolutely nothing in between, including no space and time, the chairs would be next to each other and they do not need the conventional gravity to get them moving towards each other.

    The fact that objects in the vacuum of space are separate is because there is “space” in between them and this space must have been created by some form of energy field.

    Thanking you especially for you great patience and endurance,

    Dr HW Looi
    email: looihw88@gmail.com

    1. Your statements are correct until the last one.

      “this space must have been created by some form of energy field.”

      This is both a bit illogical and also somewhat confused. A field is one thing; energy is something it can have, but there is no such thing as an “energy field”. There are electric fields; particles that are ripples in these fields are called photons and they can carry energy from one place to another. There are gravitational fields; particles (hypothetical but deeply plausible) that are ripples in these fields, essentially ripples in space itself, are called gravitons, and they too can carry energy from one place to another. But an “energy field” wouldn’t make any sense. You must mean something else.

      In general, we don’t know why there is space, or why there is anything that can be called a “universe”, but in principle it need not be created by some other field, any more than the electron or quark fields that are found throughout space need to be created by some other field. There are many theories as to how space comes to be, including theories that have time but not space at first, and then develop space through a very subtle mechanism that is far too difficult to explain here. But once you have space, you can have objects in it, and they will remain separate.

      Am I right that your real question is “why is there any such thing as space, through which waves and particles can move and in which one can find objects?” If so, the answer is not known, but it does not have anything to do with an energy field.

  202. Dear Sir,

    In the vacuum of space, if there are two bodies A and B and if there is absolutely nothing between them they should come together since there is nothing to keep them apart. They must be some form of energy field that creates the space to keep them apart. This energy field which keep the planets and stars apart could have been created during the big bang.

    Gravity could also be explained by assuming that any body with a mass will be capable of absorbing this field and creating a decreasing density of this field as we go nearer and nearer to this body.

    So before the big bang since there is really absolutely nothing, with no time and space, all virtual particles that are being continuously created out of nothing should come together and occupy the same point and as such would ultimately form a singularity with almost infinite density and triggers off the big bang.

    This could happen if there is a mechanism which separate these virtual particles from their anti-particles.
    So has anybody found such a mechanism?

    Dr HW Looi
    gmail: looihw88@gmail.com

    1. Your first point is correct, but incomplete.

      ” if there are two bodies A and B and if there is absolutely nothing between them they should come together since there is nothing to keep them apart. ”

      Two bodies A and B that are stationary relative to one another will gradually come together under the force of gravity.

      However, if they are moving relative to one another, conservation of angular momentum will drastically slow the rate at which they can come together. That is why the planets can orbit the sun for billions of years under the force of gravity.

      So there is no need for an energy field to keep objects apart and to explain why objects do not come together.

      “before the big bang since there is really absolutely nothing, with no time and space, ”

      We don’t know that.

  203. Hey Matt

    Discovered your blog over the weekend am a second year physics student at the university of south australia but all the people here are dopes and not interested in fundamental questions. I’m interested in the background independent aspect of the field theories ie: what if we throw away space time altogether and just haver the fields and their interactions? . Bothered by you’re inclusion of the “muon field” as seperate from the electron field in this discussion tho. The muon and the tau should just be other kind of excitations in the “electron field” right? the question is how many fields and how many ways to fold them? the less the better

    1. Your field question is slightly ill-posed, though probably you can refine it. A field in classical physics is a function f(x,t), with a set of differential equations that govern its behavior. If you throw away space and time it becomes a number f with a set of algebraic equations that determine it. Quantum mechanics of such a field is just ordinary integrals. This is probably not what you had in mind, but I’m not sure yet what you really did have in mind. Did you want to remove the *metric* on space and time [i.e. our ability to measure distances?] while keeping the space and the time around?

      There is strong evidence against thinking of the muon and tau as excitations in the electron field. When you excite an atom, the thing you expect is that the excited state can decay to the ground state by the emission of a photon. But muons decay to electrons only via the weak nuclear interaction, during which they spit off a neutrinos and an anti-neutrino. The process muon –> electron + photon has been searched for, but very extensive experiments have never seen this process (or the corresponding ones for taus) and the result so far is that less than 1 in 100,000,000,000 muons decays this way.

      You might still wonder whether that just means there’s something special about the way that a muon and a tau are excited forms of the electron that is different from atoms. Well, here’s more evidence. If the electron is special, in that the muon and tau are excited versions of it, then you would not expect a tau to decay to a muon (plus a neutrino and anti-neutrino) at the same rate that it decays to an electron (plus a neutrino and anti-neutrino). But in fact the rates for these two processes are the same.

      Finally, the electron and muon and tau are in some sense intrinsically massless; they only develop a mass when the Higgs field becomes non-zero (see http://profmattstrassler.com/articles-and-posts/particle-physics-basics/the-known-particles-if-the-higgs-field-were-zero/ ). So their masses are determined by how they individually interact with the Higgs field, not by some internal dynamics.

      I guess I should mention that in atomic physics there are many excited states with heavier and heavier masses, but beyond the tau there’s no sign of any fourth lepton even up to masses over 100 times larger than the tau’s mass.

      You may contrast this with the proton, for which there are many excited versions, all of which decay preferentially back to a proton plus some pions (which for proton-like objects, made from quarks, gluons and antiquarks, is an even more efficient process than emitting photons).

      Why should we have three versions of almost the same type of particle? And why would they all interact very differently with the Higgs field? There have been many, many proposals, but so far nothing is yet known about the answers to these questions.

  204. This is a great article.

    I do have a few questions though. I am studying particle physics as an undergraduate and I am struggling with this concept a bit. After immense amounts of research, I have found no explanation as to why virtual particles are needed to explain forces. What do particles offer that fields do not? Can oscillators in a field not be explained by quanitizing the field? I’ve read most all of Feynman’s lectures and many others and have found no answer.

    Also, I am a confused about the state of the virtual particles before and after the interaction between two field particles. Do they only form when approaching other field particles? Are they emitting these virtual particles at all times? What actually happens to the particle after the interaction?

    Thank you very much for this article. It has helped me very much conceptually.

      1. Not exactly. I know Green’s functions are used to solve advanced differential equations to sometime correlate functions but I have a weak background in electrodynamics and have not taken any classes on it yet. Is this where I should look to find my answer?

        1. Yes. A virtual particle is to a real particle as a Green’s function is to a wave.

          A wave is a solution to a wave equation, with zero on the right hand side of the equation. Such a wave can propagate on its own. Light is an example; it can travel through space even if there are no electric charges around. Quantum light is made from quanta [i.e. photons] and these are what we call “particles” of light.

          A Green function is a solution to the wave equation in the presence of a point source on the right-hand side. You learn how two electrons affect each other by studying the field at one electron’s location due to an electron at a second point; this involves the Green function with a point source at the location of the second electron. A simple example of a Green function is the 1/r electrostatic potential between two static electrons. The potential would never take this 1/r form on its own, if the charges were not there.

  205. Sorry if this is a double-post. I tried to comment earlier but it didn’t seem to go through (which is hopefully a good thing, because I’ve since edited the post.)

    This is all extremely fascinating—thank you!

    The impression I’m getting about the fundamental nature of matter is as follows, and I’d love to know if I’m more or less on the right track. Please forgive me if I’m way off-base; the last thing I want to do is confuse other lay-readers like myself with falsehoods!

    –Quantized fields that permeate the fabric of space-time—rather than particles—might be thought of as truly “fundamental.”

    –A “fundamental particle” is a stable ripple of contained energy on a field.

    –At least partly because of the quantized nature of fields, stable ripples in a given field (that is, all “real” particles of a given type) can’t just have any old values for their properties. Instead, they must have certain characteristics in common (mass, spin number, etc.) and are essentially indistinguishable from one another.

    –There are plenty of unstable disturbances in a field, too (more of them than stable ripples even?), which can only be very short-lived and need not behave the same way as stable ripples (particles) do in the same field. They are known confusingly as “virtual particles.”

    –Stable ripples (“particles”) and unstable disturbances (“virtual particles”) alike interact with other ripples/disturbances in their own field and/or in (some) other fields (the specifics of which other fields’ disturbances a given field can interact with vary from field to field).

    –The results of these interactions among field ripples/disturbances are matter and forces as we know them.

    If I’m basically on the right track, I have a few questions to throw at you:

    I understand that the Higgs field is thought to pervade all of space-time. Is the same true of other fields associated with elementary particles, such as the “electron field” and “photon field” you’ve mentioned? Is it thought that there is just one “electron” field that exists everywhere?

    When you speak of unstable disturbances (short-lived “virtual particles”) in a field—do these phase in and out everywhere, all the time, in every field, in something like a uniform fashion? Is this what is meant by “quantum fluctuations” in a vacuum, or “vacuum energy”? If so, is there a theory as to what “causes” these disturbances? In other words, are these disturbances always the result of the presence of other nearby particles (real or virtual) interacting with the given field? Or is this just something fields do?

    Am I right to think of a field as something like a fabric of quantized energy?

    I really appreciate what you are doing on this site. Many thanks in advance for any corrective feedback you can provide for my evolving conceptual comprehension of the nature of reality. I know I can never fully understand this stuff without the math, but I really want to try!

    1. Remarkable. Almost everything you’ve said is basically right. A small issue:

      “A “fundamental particle” is a stable ripple of contained energy on a field”

      well, the words “contained energy” aren’t really necessary and don’t really have content. Any ripple in any field has some energy. So let’s just say “A “fundamental particle” is a stable ripple on a field” and be done with it…

      … except for one additional subtlety with the word “stable”… it needs to be “relatively stable”, because most particles eventually decay to other particles, though it takes a while.

      Now, to your questions.

      Yes, all fields related to the known elementary particles are believed to exist everywhere in space, and at all times. There is just one electron field that is everywhere in the universe. What makes the Higgs field different from the other known fields is that it is non-zero on average everywhere in the universe, while, say, the photon field [i.e. the electric and magnetic fields treated together] and the electron field are on average essentially zero.

      Quantum fields are constantly fluctuating, and the unstable disturbances that we call “virtual particles” are always there. It is just something that quantum fields do, and the mathematics known as “quantum field theory”, which I teach to first and second year graduate students and which is very well established both theoretically and experimentally, does a great job of predicting the details of these fluctuations/disturbances/virtual particles. No lingering mysteries here, not for many decades.

      I don’t know what a “fabric of quantized energy” is or means. You can’t really explain fields in terms of more fundamental things, at least not at this time; as far as we know, they are the fundamental things. Fields are just the basic ingredients of our universe, in our current view. Of course this picture of the universe is likely to evolve over time as we learn more, and so my point of view may someday have to change. Right now it is consistent with all experiments.

      1. Great! Thanks for this. Conceptual explanations of this material are incredibly hard to come by, which is really a shame, since it’s endlessly interesting. Have you ever thought about writing a book on particle/field physics for laypeople? I’m sure you have real science to attend to, but it’s worth keeping in mind that there are probably very few people in the world who possess both your level of expertise in the discipline and your knack for dumbing things down juuust the right amount when explaining mind-blowing and unintuitive quantum truths to the mathematically challenged.

        Here is another one, if you find a few free moments: does QFT have implications for making sense of wave-particle “duality” and wave-function “collapse”? Given your understanding of particles as (relatively stable) ripples in constantly fluctuating fields, how do you wrap your mind around the observed effect of a single electron apparently “interfering” with itself in the double-slit experiment?

        Thanks again,
        Mike

        1. I am writing a book for laypeople. Slowly.

          Quantum field theory does not help make quantum mechanics less weird. It doesn’t make it more so…

          There are reasons I have not discussed the difficulties of making sense of a quantum mechanical world. It’s hard to do it well, and I’m not ready.

          The way I wrapped my mind around the double slit experiment is that I watched the experiment being done (with photons, not electrons, but it’s the same effect.) So then I had no choice in the matter. I’ll write about this someday.

          1. What happens if we put a double-slit experiment with an electron in a gas chamber? I assume we shall see the tracks of electrons, but no interference? Or no tracks of electrons, but with interference?

      2. Matt Strassler wrote:
        “well, the words “contained energy” aren’t really necessary”

        Are the concretized particles not ‘contained’ in that they are a resonant standing wave in the Higgs field, said resonant standing wave giving the concretized particles their mass and thus inertia? That was my understanding.

        If so, that opens up some pretty interesting possibilities for tearing invariant-mass matter down into its constituent energy…

  206. If a particle is “a nice, regular ripple in a field” would it be similar to a phonon concept in a virtual-particle “Dirac-sea”?

    1. No, what I have in mind is simpler. Think of light — an ordinary ripple in the electric and magnetic field. Now remember that in quantum mechanics that light cannot be arbitrarily dim — there is a dimmest possible flash, a ripple of lowest possible intensity. That dimmest possible flash — the ripple of lowest possible intensity — is a photon.

      Does that help?

      1. That helps a lot when discussing things like electrons: since they have the exact same charge, and—ignoring quarks and such—all electrical charge variations come in increments of one electron’s charge, it’s sort of intuitive that “an electron is the smallest possible variation of the charge field”. I can even sort of imagine moving electrons as a kind of moving ripple, and even stationary electrons (my mind throws out the analogy of a spring in a mattress oscillating without affecting its neighbors).

        But I don’t really get the “ripple of lowest possible intensity” analogy for photons. As far as I can tell, you can have photons of pretty much every frequency, from ultra-long-wave radio to ultra-hard gamma rays, and I can’t figure out anything that’s “smallest” and they all share, the way charge is for electrons. Is that spin or something?

        (I get that *specific* interactions, like electron excitation in atoms, produce or absorb photons of a certain energy, but unless I’m very much mistaken that’s quite a different issue.)

      2. I get that explanation for electrons, or at least it feels intuitive; you can only have variations of charge in specific units, and those are electrons (and positrons), at least until you start wondering about quarks and why they’re allowed to use thirds. But unless I’m very much mistaken, you can have photons of pretty much every energy, from ultra-long-wavelength radio to hard gamma rays. What exactly is the quantity a photon is “the least intensity ripple” of?

        1. A photon with frequency f is the smallest possible intensity of frequency-f light, i.e. the smallest possible frequency-f ripple in the electromagnetic field. “Intensity” here is basically the same as energy: the amount of work your light could do. “Frequency” is basically the same as color.

          1. Do we have any indication whether or not real photons can become virtual? I know Chalmers University in 2011 concretized virtual photons into real photons using a SQUID, but I’ve not heard of the opposite occurring.

            Take, for instance, a virtual photon of 1 micron wavelength. It gets red-shifted (say, via universal expansion) to 2 micron wavelength. Since E=hf (E=hc/wavelength) and f has decreased, E has likewise decreased.

            If a real photon is the minimum possible excitation (a single quanta) for a given frequency in the EM field, and E decreases, is it possible for a real photon to become virtual, or is the relationship between E and f such that a concretized photon always remains concretized regardless of blue- or red-shift?

  207. If a particle is “a nice, regular ripple in a field”, how would you describe a string from string theory? What will they appear like in the field?

    1. Well, a string is a ripple in a string-field, something for which we have no daily intuition. We have intuition for fields because we live in a wind field, a temperature field, a air-density field, and so forth, and wind, sound, heat diffusion are all things we can imagine and study rather easily. But a string field actually contains an infinite number of types of ordinary fields — and we don’t encounter one in daily life — so you can imagine building an intuition for that would not be easy. Let me think about how I might explain this one someday — this will not be coming soon.

      1. What are the grounds for calling this ripple a “string”, in contrast to a point-like “particle”?

        1. More or less what you’d think: if you strike it, you will find an elementary string can respond in all sorts of ways — by wiggling in all sorts of different configurations — while an elementary particle cannot do that.

  208. Once again, brilliant. I understand so much more about particle fields after reading this article.

  209. Pingback: Quora
  210. In your article “What’s a proton?” you state that there are “zillions of gluons, antiquarks, and quarks in a proton”. Do most of these qualify as virtual particles as described in this article, i.e. as mere disturbances of their fields that are in many respects quite unlike real particles?

  211. As an electrical / electronics engineer with a strong interest in physics (but not great at advanced math) your articles have explained more to me in a few hours than in the several years I spent in secondary and tertiary education. You write and diagram with amazing clarity. I do notice that when questions are asked about speculative physics (such as the possibility of extra dimensions), you confine your responses to answers that can now (or may be in the near future) subject to experiment.

    Your explanation of “virtual particles” here eliminates most of my confusion about articles on the subject I have read elsewhere, but two bits of confusion persists; if “virtual particles” are complex disturbances in a field, then, can the disturbances (through Fourier transforms) be described as an agglomeration of resonances (particles)? also, what did / does Prof. Hawking mean by HIS use of the term in relation to what happens at the event horizon of a black hole? (I know I should ask Prof. Hawking, but he is somewhat less accessible :-))

    1. [quote]You [Strassler] write and diagram with amazing clarity. I do notice that when questions are asked about speculative physics (such as the possibility of extra dimensions), you confine your responses to answers that can now (or may be in the near future) subject to experiment.[/quote]

      Well, I am asking, has there been to date [ in my location, April 19, 2013, Monday, 10:48 AM ) experimentation on the reality in observation of virtual particles, or virtual particels are all speculative?

  212. I’m confused as to what explains the ‘vacuum energy’ where there are no real particles disturbing any fields ? what powers the cosmological constant ?

    1. There’s plenty of energy in those disturbances. You seem to be assuming that the only way fields can carry energy is through their particles, but this is not the case. Analogy: one way for a spring to carry energy is to go back and forth at its resonant frequency, but a spring could carry energy even if you pushed it back and forth in an arbitrary way. And a quantum spring, thanks to the uncertainty principle, will move around, in a limited but arbitrary way, and with non-zero energy, even if you just leave it alone. The difference in the math is the same for particles (resonant behavior that can go on its own) versus general disturbances (which require an outside push, or just the jiggling assured by the uncertainty principle.)

  213. Is this “virtual particle=>causality issue” maybe coming from discussions like those in Bjorken and Drell section 12.3 ? My reading of their conclusion is that there isn’t really a problem though.

  214. My lay understanding was that virtual particles “challenge” conventional notions of cause and effect, but you use the word “cause” in very conventional ways in this article.

    1. Could you try to help me figure out where this notion comes from? Do you know where you read it? I have some guesses as to where this conception comes from, but I wonder whether there are modern books promulgating the idea. While it is true that one has to be careful in general about assuming that all processes can be described in terms of cause and effect (even before accounting for quantum mechanics), and also true that quantum mechanics is weird, no doubt about it , there is no profound challenge to basic causality in this context. Certainly I do not think you will not find any discussion of challenges to causality from “virtual particles” (i.e. generalized disturbances in fields) in any modern quantum field theory book.

  215. A wonderful picture of the world on “Quantum field theory ”, recall wave-particle duality ,what about “Quantum entanglement ”?

  216. Thanks for the nice article. When reading about Hawking radiation, one is told that when a pair virtual particles is created near the event horizon, one may fall into the black body and the other escape as radiation. Are the virtual particles in this situation somewhat different than the ones you describe? Do they become “real” particles in this case?

    1. Great question. If the escaping virtual particle becomes real then there would be an increase in the mass of the observable universe. The virtual particle absorbed by the black hole should also become real and should then increase the mass of the black hole and not reduce its mass. The information regarding the absorbed particle is not lost as the escaping particle reflects the information of the absorbed particle as virtual particles are created in opposite pairs.
      vlrmisc@aol.com 17Dec2019

  217. If I’m understanding correctly, your saying virtual ‘particles’ arise because of ‘nearby’ real particles disturbing a field, those disturbances being the virtual ‘particles’. That left me wondering what then explains the ‘vacuum energy’ of empty space? I naively thought virtual ‘particles’ where popping in and out of existence with no real particles nearby. I took it that the cosmological constant was somehow ’powered’ by vacuum energy ?

  218. One of the things that tripped me up as a student, was also just how arbitrarily small you could make the difference between real and virtual.
    Usually the definition of a virtual particle is a ‘thing’ that does not satisfy the usual energy-momentum rules, or the ‘thing’ which is an internal leg in a Feynman diagram.

    But then every particle we have ever observed is an internal leg of a bigger Feynman diagram. For instance, the electron we measured from a particle accelerator eventually is absorbed by an atom somewhere, and hence becomes an internal leg in that diagram. So then most people say, well its just a question of lifetime. Long lived particles are called real, short lived particles are virtual.

    But then suppose you have a photon that was emitted from the Pleiades, and it presumably is real (or almost real) all the way until it is absorbed in your retina. So have you just measured a real or virtual photon? It sorta depends on how you draw the diagram!

    1. You are absolutely correct to point out this subtlety! It is, in the end, a matter of degree.

      In general, what we have in quantum fields are disturbances of many types. There is a very special disturbance we may call a particle, which is a ripple that can in principle travel forever. But this is an idealization: any real particle interacts with other objects, and this means nothing is ever exactly this precise, idealized ripple. So the issue is how close is it to the ideal case. In most physical processes one deals with objects that are clearly either close to the ideal or very far from the ideal. A photon traveling from the Pleiades is clearly about as close as you are going to get to the ideal; its energy and momentum are almost the perfect match that you would expect for a massless particle. The disturbance between [a “virtual photon exchanged”] between an electron and a nucleus in an atom has very little energy and a lot of momentum; it is very far from what you would call a particle.

    2. Columbia, your point here about every particle being part of a larger diagram suggests a way to dispel puzzlement about EPR-type experiments. The EPR “paradox” appears, I think, only when there are humans (or other sentient beings) acting as scientists, creating the experimental conditions and collecting the results. We like to think of ourselves as standing outside the experiment, causing photons to become entangled, then sending them off to distant observing stations where randomizing polarizers have been set up in certain ways, such that the results will be automatically recorded at each station by appropriate detectors, then later transmitted from place to place and compared with each other, and so on.

      But we (the scientists) are part of an inconceivably tangled up arrangement of diagrams within diagrams, in which events “just happen” — including all of our own actions. If we could examine all those linked diagrams and the evidence of what the results had been, without ideas about free will and intention, it seems to me that there would be no place in the description of what happened for any paradoxes to be inserted.

  219. So what you are saying essentially is that transient ripples, which are not stable or sustainable, are caused by every particle in every other field with which they can interact. If another particle is present, their ripples combine and the particles DO interact. These ripples look in *some* ways like the ghost of a particle in that field, enough that we can use the equations of particles to calculate effects. But in other ways, including their stability, they’re not really like particles at all.

    Okay. I got everything except how “virtual particles” are involved in particle CREATION. At some point, wouldn’t the transient ripples have to coalesce into wave(s) that ARE stable?

    1. That’s basically right. I wouldn’t use the term “ghost” (for two reasons, one being that ghost is used as a technical term elsewhere, but more importantly that the relation is really that disturbances are the more general case, while a particle is an extremely special form of disturbance.)

      How “virtual particles” can create something: the transient ripples themselves are still not stable in this case. They fall apart, into true particles.

      For example, if the two electrons in my picture came together with enough energy, the transient ripple shown in the figure could have enough energy to produce an electron-positron pair (so that there would be four particles in the final part of the picture, three electrons and a positron.) The disturbance in the field can carry lots of energy, and that energy can be turned into particles. But the disturbance itself is still transient. Does that answer the question?

      1. Hold on, I thought transient ripples weren’t necessarily a result of other disturbances, and can arise just due to the uncertainty principle. Is this true? The original asker seemed to purport this

  220. In dense aether theory the water surface can serve as a low dimensional analogy of 4D space-time and the density fluctuations of the underwater are the analogy of virtual particles, after then.

  221. Hi,

    I have some questions about the meaning(s) of the word “particle” in the field of high energy physics:

    1-What is the safest way to call a quantum object: an elementary particle? or a quantum particle? or a quantum wave? or we just call them by their names like electrons or neutrinos..etc? or the question is just meaningless?

    2-What is the meaning of the word particle in “the branch of particle physics”?

    3-Which is more fundamental, the quantum particle, or the quantum field? (I did not find consensus in this one)

    4-And finally, what confuses me most, why “elementary particles” are irreducible representations of the Poincare group? and what is the meaning of the word particle in this context?

    Sing

    1. 1) A name’s a name (see Shakespeare) — what matters is not what you call it but whether you understand it… fair? As long as you understand the object does not correspond to any concept in English or in daily life, and that it has some properties that you just have to learn about, you can call an electron an elementary particle, a quantum of the electron field, or a quantized wave in the electron field. Elementary particle is rather short and the least mysterious sounding. “Quantum” is shortest and most accurate, but sounds very mysterious.

      2) You mean, what does “particle physics” mean when referring to it as a subject of study? Just that many experiments on the basic properties of nature require studying its particles, and if you do those experiments, propose them, study them, interpret them, etc., you’re doing “particle physics.”

      3) Field is more fundamental. Not all fields have particles, while all particles are quanta of fields. That’s why the equations used to describe the physics of the standard model are called quantum field theory, not quantum particle theory. [An example of a quantum field theory that has no particles is a “conformal field theory”, very important in studying matter at a phase transition, such as a magnet at the critical temperature where it loses its magnetization.]

      4) Start with classical physics (no uncertain principle) to keep things simple. Then a massive particle is a localized object with a definite mass, energy, momentum and position, and orientation. Let’s put one in front of you, at rest. I can now generate a representation of the Poincare group — the group of all translations, rotations, and changes of reference frame (boosts with a constant velocity) — by simply moving the particle to a new position, rotating the particle, or looking at the particle from a different frame. That set of particle states (with all those different positions, orientations, and velocities) form a complete representation of the Poincare group.

      The only thing different in quantum mechanics is that I can’t precisely localize the particle at the same time I precisely determine the momentum and energy. But I can do a pretty good job on both, or a perfect job on either one. This complication doesn’t change the argument that the full set of things that a particle can do form a representation of the Poincare group.

      For a massless particle I can’t start with a particle at rest, but the basic strategy for finding the full representation is the same.

  222. Thanks for a great and understandable explanation.

    Is it correct to say that these quantum fields pervade spacetime – or is it better to say the set of fields actually composes (creates?) spacetime? Also, what is the complete list of fields currently known?

    1. Most fields are best thought of as pervading three-dimensional-space and time, except for the graviton field, also known as the “metric” of space-time (the object that is needed to decide how far part two points are) which is really intrinsic to space-time.

      In most theories with extra dimensions, some of the fields that we observe would actually form a part of the metric of the higher-dimensional spacetime. In other words, one explanation as to why there are so many fields in nature might be that we live in a world that has some of its dimensions wrapped up (think of how a hose has a large dimension along the hose and small dimension around the hose) and that the metric of the full space-time looks to us, in three-dimensional space, like a metric for three-dimensional space and time along with many other fields whose explanation seems non-obvious.

      A complete list of fields is ill-defined, but I can give you the list of apparently-elementary fields. [There are many non-elementary fields too, including the proton field; just as a proton is composite made from quarks, antiquarks and gluons, so is its field made from other fields. And the wind field and temperature fields in air, or a density field in a metal, are composite too.] The list of known apparently-elementary fields is essentially just the list of known particles, http://profmattstrassler.com/articles-and-posts/particle-physics-basics/the-known-apparently-elementary-particles/, or even more completely, the list of particles before the Higgs gets a non-zero value, http://profmattstrassler.com/articles-and-posts/particle-physics-basics/the-known-particles-if-the-higgs-field-were-zero/ (plus the graviton field, which I didn’t put on the slides.)

Leave a Reply to Shawn HalaykaCancel reply

Search

Buy The Book

A decay of a Higgs boson, as reconstructed by the CMS experiment at the LHC